Archer Cardiovascular

¡Supera tus tareas y exámenes ahora con Quizwiz!

A patient recovering from myocardial infarction is presenting with heart rate 110 beats per minute, blood pressure 86/58 mmHg, crackles, shortness of breath, dusky skin, and jugular vein distention. Which action should the nurse recognize as the highest priority? A. Administer medications to increase stroke volume. B. Provide analgesics. C. Obtain a STAT electrocardiogram and troponins. D. Administer fluid replacement to increase blood pressure. Submit Answer

Explanation Choice A is correct. Based on the assessment information, the nurse can determine the patient is experiencing cardiogenic shock secondary to myocardial infarction. Since cardiogenic trauma occurs as a result of the heart not pumping effectively, the highest priority is to increase cardiac output to ensure adequate tissue perfusion. Cardiac Output = Stroke volume x Heart rate. Medications that improve stroke volume will improve cardiac output in cardiogenic shock. The following agents may be used in the pharmacological management of cardiogenic shock. Inotropes: Positive inotropes strengthen the heart contractility (increase stroke volume). Dobutamine has more beta-adrenergic action than alpha activity. It causes peripheral vasodilation while increasing contractility. But in higher doses, it may increase heart rate and exacerbate myocardial ischemia. Vasopressors: In severe shock, vasopressors (Dopamine, Norepinephrine) maintain blood pressure but decrease blood flow to organs. They increase afterload and reduce cardiac output. However, they may be needed initially to provide hemodynamic support. Dopamine increases myocardial contractility and maintains blood pressure. If dopamine fails to support blood pressure, norepinephrine is added. Vasodilators: Vasodilators (Nitroglycerin) decrease venous return (preload) to the heart and decrease peripheral resistance (afterload). Although vasodilators may drop blood pressure, they sustain cardiac output and help achieve hemodynamic stability when combined with vasopressor support in cardiogenic shock. Supplemental oxygen may also be necessary to increase tissue oxygenation. Choice B is incorrect. There is no assessment information in the question that points to chest pain. If a patient in cardiogenic shock is showing signs or complaining of pain, this action would be appropriate, but not the highest priority. Choice C is incorrect. The patient recently experienced MI, so they should already be on a telemetry monitor. ECG will likely be abnormal and troponins may still be elevated. This action may be appropriate but will not change the immediate treatment of shock, so it would not be the highest priority. Choice D is incorrect. Fluid replacement is not the correct immediate action because the patient is showing signs of pulmonary edema (crackles, shortness of breath, jugular vein distention). Cardiac output needs to be improved before considering the additional fluid volume. This action might be appropriate if the patient was in hypovolemic shock, not cardiogenic. NCSBN Client Need Topic: Establishing priorities, illness management, medical emergencies, pathophysiology Additional Info Source : Archer Review Last Updated - 09, Feb 2022

The nurse is caring for a patient with left-sided heart failure. Which of the following signs and symptoms is related more to right-sided heart failure? A. Ascites B. Tachypnea C. Cough D. Crackles and wheezes Submit Answer

Explanation Choice A is correct. Ascites is a symptom of right-sided heart failure, not left-sided. Right-sided heart failure involves congestion in the systemic circulation. Patients with right-sided heart failure may also experience jugular vein distention, oliguria, weight gain, and peripheral edema. Choice B is incorrect. Tachypnea, or more frequent than normal respirations, is seen in left-sided heart failure as breathing becomes more difficult. In left-sided heart failure, fluid backs up into the lungs and makes breathing more difficult. Choice C is incorrect. A cough, along with other heart failure symptoms, is a sign of left-sided heart failure. As fluid backs up in a patient's lungs, the patient may present with a cough. Choice D is incorrect. Crackles and wheezes upon respiratory auscultation are a sign of left-sided heart failure. As fluid backs up into the lungs because the heart is unable to pump properly, the lungs sound wet, wheezy, and may present with crackles. NCSBN client need Topic: Physiologic integrity, alterations in body systems Additional Info

The nurse is caring for a patient post-coronary artery bypass graft (CABG). The nurse is implementing measures to address the nursing diagnosis, "ineffective airway clearance related to retained secretions and excess secretions." Which nursing intervention is the least appropriate? A. Administering aspirin prior to deep breathing and coughing exercises. B. Splinting the incision site with "heart pillows" or pillows before coughing. C. Assisting the client to ambulate as tolerated. D. Teaching the client the correct use of an incentive spirometer. Submit Answer

Explanation Choice A is correct. Aspirin is an NSAID that has an antiplatelet property. Administering aspirin to the patient may put the patient at risk for postoperative bleeding. The patient needs opioids to manage his pain, not aspirin. Choice B is incorrect. Splinting the incision site before coughing promotes more intense coughing efforts. Therefore, it is an appropriate nursing intervention. Choice C is incorrect. Respiratory effort is increased with ambulation; therefore, deep breathing occurs, helping to clear chest secretions. Choice D is incorrect. The correct use of an incentive spirometer encourages sustained inspiration to open alveoli, promoting the clearing of chest secretions. Last Updated - 15, Nov 2021

The nurse is caring for a client with cardiac tamponade. Which vital signs are expected? A. HR: 109 bpm; RR: 26; BP: 88/71 mmHg B. HR: 90 bpm; RR: 32; BP: 90/52 mmHg C. HR: 115 bpm; RR: 22; BP: 140/78 mmHg D. HR: 54 bpm; RR: 14; BP: 161/52 mmHg Submit Answer

Explanation Choice A is correct. Classic manifestations of cardiac tamponade include tachycardia, tachypnea, jugular venous distention, and hypotension with a narrowed pulse pressure. Choices B, C, and D are incorrect. Cardiac tamponade would typically cause features of tachycardia, hypotension, tachypnea, and a narrowed pulse pressure. Additional Info Cardiac tamponade may be caused by an array of infectious and noninfectious reasons. Immediate treatment of cardiac tamponade would be pericardiocentesis. A needle is inserted to aspirate the pericardial fluid in this ultrasound-guided procedure. The provider may elect to leave a temporary catheter in place in the pericardium to drain more fluid. Nursing care involves reporting any suspicion of cardiac tamponade promptly to the provider. Last Updated - 28, Apr 2022

Two nurses are taking an apical-radial pulse and note a difference in the pulse rate of 8 bpm. The nurse would document this difference as to which of the following? A. Pulse deficit B. Pulse amplitude C. Ventricular rhythm D. Heart arrhythmia Submit Answer

Explanation Choice A is correct. Counting of the pulse at the apex of the heart and at the radial artery simultaneously is used to assess the apical-radial pulse rate. A difference between the apical and radial pulse rates is called the pulse deficit, which indicates that all of the heartbeats are not reaching the peripheral arteries or are too weak to be palpated. When taking a pulse, the rate, rhythm, and strength or amplitude of the pulse are noted. The average pulse in an adult is between 60 and 100 beats per minute. The rhythm is checked for possible irregularities, which may be an indication of the general condition of the heart and the circulatory system. Choice B is incorrect. Pulse amplitude is defined as the strength of a pulse. It is often described as nonpalpable, weak, thready, secure, or bounding. Choice C is incorrect. Ventricular rhythm relates to the rhythm with which the ventricles contract and relax within a cardiac cycle. Choice D is incorrect. Heart arrhythmia, also known as irregular heartbeat or cardiac dysrhythmia, is a group of conditions where the pulse is intermittent, too slow, or too fast. NCSBN Client Need Topic: Physiological Integrity, Subtopic: Physiological Adaptation; Pulse Last Updated - 18, Dec 2021

The nurse is caring for a client with congestive heart failure (CHF). The nurse should anticipate a prescription for which medication? A. Enalapril B. Verapamil C. Lovastatin D. Gemfibrozil Submit Answer

Explanation Choice A is correct. Enalapril is an ACE inhibitor and this drug class is indicated in the treatment of heart failure to prevent ventricular remodeling. Choices B, C, and D are incorrect. Verapamil is a calcium channel blocker, which is contraindicated in the management of heart failure because of its adverse effects on cardiac output. Lovastatin and gemfibrozil are medications used to reduce cholesterol and are not directly used in the management of heart failure. Additional Info For a client with heart failure, self-management strategies include following the MAWDS approach. Medications • Take prescribed medications, including ACE-I's, diuretics, and low-dose beta-blockers. • Avoid NSAIDs to prevent sodium and fluid retention. Activity • Stay as active doing aerobic activities. Weight • Weigh each day at the same time on the same scale to monitor for fluid retention. Report a weight gain of 1-2 pounds overnight; 3-5 pounds in a week Diet • Limit daily sodium intake to 2 to 3 g as directed. Symptoms • Note any new or worsening symptoms and immediately notify the health care provider. Source : Archer Review Last Updated - 02, Jan 2022

The nurse is taking care of a client that is 24 hours post-angioplasty. The client says, "I don't feel good today. I don't feel like eating." What is the nurse's best action? A. Ask what the statement means to the client. B. Delegate an LPN to assess the client. C. Notify the physician. D. Encourage the client to eat. Submit Answer

Explanation Choice A is correct. The nurse should assess the client first before implementing any intervention. Asking what the client means by his statement explores the client's feelings and provides information regarding his condition. Choice B is incorrect. The LPN cannot assess the client. It is the RN's responsibility to perform an assessment. Choice C is incorrect. The nurse should assess the client first before implementing any intervention. The nurse will need additional information to convey to the physician regarding the client's condition. Choice D is incorrect. The nurse should assess the client first before implementing any intervention. Last Updated - 28, Dec 2021

A nurse is providing discharge instructions to a Chinese-American client regarding dietary modifications. As the nurse reviews the discharge instructions, the client continuously turns away from the nurse. Which of the following is the most appropriate response by the nurse? A. Continue with the instructions, verify client understanding, and let the client ask questions B. Walk around the client so that the nurse constantly faces the client while maintaining eye-to-eye contact C. Give the client a dietary booklet and let him know you will return later to continue with the instructions D. Emphasize to the client that it is imperative he provides his full attention during the provision of the discharge instructions Submit Answer

Explanation Choice A is correct. Most Chinese Americans maintain a formal distance from others as a form of respect. Often, Chinese Americans are uncomfortable with face-to-face communication, especially when eye contact is direct. Direct and prolonged eye contact is avoided in Chinese culture, as doing so is meant to convey anger and challenge the other person. Choice B is incorrect. If a Chinese-American client turns away from the nurse during the provision of discharge instructions, the nurse should not attempt to force the client to engage in face-to-face or direct eye contact. Such attempts would be considered aggressive and unprofessional. Choice C is incorrect. Although providing the client with a booklet is an excellent manner to reinforce client education following discharge teaching, this does not in and of itself take the place of providing the client with oral discharge instructions regarding dietary modifications. Choice D is incorrect. Emphasizing to the client that it is imperative he provides his full attention during the provision of the discharge instructions would likely be viewed as condescending and, as such, should be avoided. Learning Objective Recognize that eye-to-eye contact is perceived as aggressive in Chinese culture and should therefore be avoided in Chinese-American clients. Additional Info The Chinese people view eye contact as a necessary tool, but not in the same way which other cultures do. In China, people make eye contact when they are angry. Eye contact is meant to challenge the other person in Chinese culture and is seen as a sign of disrespect. Last Updated - 14, Sep 2022

A client with a history of long-standing hypertension and hyperlipidemia is complaining of shortness of breath and weakness in the legs. Which of the following may be occurring? A. The patient may be having a heart attack. B. The patient is exhibiting signs of pulmonary embolism. C. The patient may be experiencing the onset of peripheral vascular disease. D. The patient is developing chronic obstructive pulmonary disease. Submit Answer

Explanation Choice A is correct. Myocardial infarction (MI) may present with symptoms of shortness of breath and muscle weakness. Silent MI is something that may not occur with pain. Other symptoms of myocardial infarction include: Chest discomfort. Most heart attacks involve pain in the center of the chest that lasts more than a few minutes - or it may go away and then return. It can feel like uncomfortable pressure, squeezing, fullness, or pain. Discomfort in other areas of the upper body. Symptoms can include pain or discomfort in one or both arms, back, neck, jaw, or stomach. Shortness of breath can occur with or without chest discomfort. Other signs include breaking out in a cold sweat, nausea, or lightheadedness. Since hyperlipidemia has no symptoms, it can cause damage before an individual realizes there is a problem. It can cause atherosclerosis and limit blood flow, increasing the risk of heart attack or stroke. Factors that can increase your risk of bad cholesterol include a poor diet, including foods that are high in saturated fat (found in animal products), and trans fats (found in some commercially baked products) can contribute to an elevated cholesterol level. Additionally, obesity, lack of exercise, age, and history of diabetes can increase the chances of experiencing hyperlipidemia. Hypertension is often referred to as "the silent killer" because some people do not experience any significant symptoms until their blood pressure is extraordinarily elevated or if symptoms (such as headache, palpitations, and dizziness) become bothersome. It is a significant public health problem and an essential area of research due to its high prevalence as well as a significant risk factor for cardiovascular diseases and other complications. The combination of hypertension and hyperlipidemia can result in several harmful health consequences. Choice B is incorrect. While a pulmonary embolism (PE) can present with shortness of breath, it is essential to correlate the answers with risk factors present in the question (hypertension and hyperlipidemia). Hypertension and hyperlipidemia are not necessarily risk factors for PE. Choice C is incorrect. While peripheral vascular disease can present with weakness and leg pain, it does not cause shortness of breath. Choice D is incorrect. While COPD can present with shortness of breath, it is not associated with hypertension and hyperlipidemia as risk factors. NCSBN Client Need Topic: Physiological Integrity; Subtopic: Physiological Adaptation Additional Info Source : Archer Review Last Updated - 15, Nov 2021

The patient recovering from cardiac surgery is wondering when he can resume sexual activity. The nurse would be most correct in stating that sexual intercourse may be returned at which point in time? A. After exercise tolerance is assessed B. One week after surgery C. When the patient can comfortably jog two miles D. Three months after surgery Submit Answer

Explanation Choice A is correct. Patients who have undergone cardiac surgery should have their exercise tolerance evaluated by a physician before resuming sexual activity. Many physicians agree that a patient may return to sexual activity if they can climb two flights of stairs without symptoms. Choice B is incorrect. All patients recover differently. Therefore a time frame is not the best way to evaluate readiness for sexual activity. Choice C is incorrect. Jogging two miles may be strenuous even for patients who have not undergone cardiac surgery. This is not the best way to judge readiness for sexual activity. Choice D is incorrect. All patients recover differently. Therefore a time frame is not the best way to evaluate readiness for sexual activity. NCSBN client need Topic: Physiological Integrity, physiological adaptation Last Updated - 15, Feb 2022

A client has been placed on a sodium-restricted diet following a myocardial infarction. Which of the following would be the most appropriate meals to suggest? A. Turkey, 1 fresh sweet potato, 1/2 cup fresh green beans, milk, and 1 orange. B. Broiled fish, 1 baked potato, ½ cup canned beets, 1 orange, and milk. C. Canned salmon, fresh broccoli, 1 biscuit, tea, and 1 apple. D. A bologna sandwich, fresh eggplant, 2 oz fresh fruit, tea, and apple juice. Submit Answer

Explanation Choice A is correct. People with heart failure may improve their symptoms by reducing the amount of sodium in their diet. Sodium is a mineral found in many foods, especially salt. Overeating salt causes the body to keep or retain too much water, worsening the fluid buildup. Patients should be encouraged to follow a low-sodium diet to help manage symptoms of hypertension and to reduce edema. One of the most natural things a patient can do at home is to reduce the amount of sodium intake. They can also eat fresh vegetables rather than canned. If canned vegetables are the only option, the patient should rinse the plants with clean water and cook them with unsalted water. Choice B is incorrect. Canned vegetables should be avoided. Choices C and D are incorrect. Canned or processed meats are higher in sodium and should be avoided. NCSBN Client Need Topic: Physiological Integrity, Subtopic: Basic Care and Comfort Last Updated - 15, Dec 2021

The nurse is caring for a client who has developed cardiac tamponade. Which of the following prescriptions should the nurse clarify with the primary healthcare provider (PHCP)? A. Positive pressure ventilation B. Pericardiocentesis C. Echocardiography D. 0.9% saline bolus Submit Answer

Explanation Choice A is correct. Positive pressure ventilation (PPV) would be detrimental to a client experiencing cardiac tamponade. This order requires follow-up. PPV increases intrathoracic pressure, which decreases venous return to the heart. This reduction of venous return impairs ventricular filling and decreases cardiac output. This would be detrimental in a cardiac tamponade where the cardiac output is already impaired. Choices B, C, and D are incorrect. A pericardiocentesis is a primary treatment for cardiac tamponade. Echocardiography is the primary way of identifying cardiac tamponade. This non-invasive approach determines the severity of the tamponade and any other cardiac structure impairment. 0.9% saline bolus may be used to temper the hypotension caused by this condition. This is not an absolute treatment but is useful in mitigating hypotension. Additional Info Cardiac tamponade is compression of the myocardium by fluid that has accumulated around the heart; this compresses the atria and the ventricles, prevents them from filling adequately, and reduces cardiac output. Cardiac tamponade may be caused by an array of infectious and noninfectious reasons. Classic manifestations of cardiac tamponade include tachycardia, hypotension, jugular venous distention, pericardial rub, and pulsus paradoxus. Last Updated - 26, Apr 2022

The nurse is assessing a client with clinical manifestations of right ventricular heart failure (HF). Which of the following statements by the client would be consistent with this diagnosis? A. "I notice that my feet are always swollen." B. "I can't seem to get rid of this wet cough." C. "I develop shortness of breath after I walk a few feet." D. "My legs start to burn if I walk long distances." Submit Answer

Explanation Choice A is correct. Right-sided heart failure manifests with peripheral edema, hepatosplenomegaly, jugular venous distention, and oliguria. The client states that his swollen feet would be consistent with right-sided heart failure. This is because of the fluid backing up into the client's body. Choices B, C, and D are incorrect. A wet cough, tachypnea, and respiratory symptoms would be consistent with left-sided heart failure. If the client reports that their legs burn with ambulation, this is suggestive of potentially something associated with peripheral arterial disease. Additional Info Last Updated - 27, Dec 2022

The nurse is caring for a patient with weak pedal pulses, absent hair on bilateral legs, and a full-thickness wound on the right lateral malleolus with defined margins including a minimal amount of serous exudate. Which of the following interventions is contraindicated for this patient? A. Apply TED hose to bilateral legs B. Assess the need for smoking cessation C. Physical therapy consult D. Obtain Ankle-Brachial Index (ABI) with a hand-held Doppler Submit Answer

Explanation Choice A is correct. The patient is presenting with signs of arterial insufficiency. The application of compression (TED hose) to the extremities is contraindicated in cases of severe arterial problems and should not be applied until cleared by the healthcare provider. The physician may want to make sure that the perfusion is adequate before clearance is given to apply a compression device. Choices B, C, and D are incorrect. These are appropriate and indicated for a patient with peripheral arterial disease (PAD). Smoking is a significant risk factor for developing arterial problems. Patients with PAD should be counseled to stop smoking (Choice B). Exercise improves the PAD symptoms. Most PAD patients do not get enough exercise; nurses should encourage patients to participate in physical therapy and to ambulate frequently throughout the day (Choice C). The ankle-brachial index (ABI) is a non-invasive way to calculate the relative severity of PAD (Choice D). NCSBN Client Need Topic: Cardiovascular, Subtopic: Potential for complications from health alterations, system-specific assessments, pathophysiology Last Updated - 21, Jan 2022

The nurse is caring for a client with the following tracing on the electrocardiogram (ECG). The nurse should anticipate a prescription for which medication? See the image below. A. Adenosine B. Atropine C. Labetalol D. Amiodarone Submit Answer

Explanation Choice A is correct. The tracing reflects supraventricular tachycardia (SVT). The preferred medication for individuals experiencing SVT includes the rapid administration of adenosine followed by a rapid flush of 0.9% saline. Adenosine slows the electrical conduction time through the AV node. Choices B, C, and D are incorrect. Atropine is indicated for the treatment of symptomatic sinus bradycardia. Labetalol is indicated for a hypertensive emergency and sinus tachycardia. While labetalol lowers heart rate, it would not treat the underlying cause of SVT. Amiodarone may be utilized for refractory SVT, but this drug is not the initial drug of choice for SVT. Amiodarone is a preferred drug for AFIB and VTACH. Additional Info During SVT, P waves may not be visible, because the P waves are embedded in the preceding T wave. A client with SVT may be asymptomatic. If the client is symptomatic, they may exhibit manifestations such as palpitations, dizziness, dyspnea, and nervousness. Treatment includes vagal maneuvers. Vagal maneuvers include having the beardown, blowing through a straw, having the primary healthcare provider (PHCP) perform a carotid massage, and, if the client is an infant, applying a bag filled with ice and water to the face above the nose and mouth for 15 to 30 seconds. If that is not effective, another vagal maneuver would be pressing the infant's knees to the chest for 15-30 seconds. If these measures are ineffective, the nurse should prepare to administer the prescribed adenosine by rapid intravenous push (IVP) that is followed by a flush of 0.9% saline. When adenosine is administered, the emergency (code) cart should be nearby, and the nurse should always have additional personnel in the room. Last Updated - 27, Jun 2022

A client arrives at the emergency department (ED) complaining of substernal chest pain. An ECG shows ST-segment elevation, and the client's cardiac troponin level is found to be elevated. Which of the following should be the nurse's primary focus? A. Reducing cardiac workload and improving myocardial oxygenation B. Confirming the suspected diagnosis and preventing complications C. Pain relief and reduction of anxiety D. Providing a quiet, non-demanding environment and reducing anxiety Submit Answer

Explanation Choice A is correct. This client displays signs and symptoms of an ST-segment elevation myocardial infarction (STEMI). In a STEMI, myocardial necrosis is occurring, with the client exhibiting ECG changes showing ST-segment elevation not quickly reversible by nitroglycerin administration. The primary focus should be on improving myocardial oxygenation and reducing cardiac workload, as these measures will reduce further expansion of the myocardial necrosis. Choice B is incorrect. Based on Maslow's hierarchy of needs, the needs to confirm the suspected STEMI diagnosis and prevent complications are not prioritized over the need to improve myocardial oxygenation and reduce the cardiac workload. Choice C is incorrect. Based on Maslow's hierarchy of needs, pain relief and anxiety reduction are not prioritized over the need to improve myocardial oxygenation and reduce the cardiac workload. Choice D is incorrect. Providing a quiet, non-demanding environment to reduce anxiety is not a nursing intervention that should be prioritized above the need to reduce cardiac workload and improve myocardial oxygenation based on Maslow's hierarchy of needs. Learning Objective Recognize that when a client presents to the emergency department with symptoms indicative of a myocardial infarction, the nurse should prioritize improving myocardial oxygenation and reducing cardiac workload according to Maslow's hierarchy of needs. Additional Info An acute myocardial infarction (MI) is myocardial necrosis resulting from acute obstruction of a coronary artery. Diagnosis occurs via ECG and cardiac markers. Immediate treatment typically includes oxygen, antianginals, antiplatelet drugs, and anticoagulants. For those with an ST-segment elevation MI (STEMI), these clients should undergo an immediate angiography with percutaneous coronary intervention (PCI); if an immediate PCI is not unavailable, administer fibrinolytics. For clients with non-ST-segment elevation MI who are stable, perform an angiography within 24 to 48 hours; for those who are unstable, an immediate angiography with PCI should be performed. Following recovery, clients should be initiated or continued on antiplatelet medications, beta-blockers, angiotensin-converting enzyme inhibitors, and/or statins. Last Updated - 30, Oct 2022

Which of the following statements correctly outlines the proper flow of blood through the heart? A. Superior and Inferior vena cavas → Right atrium → Tricuspid valve → Right ventricle → Pulmonary valve → Pulmonary artery → Lungs → Pulmonary veins → Left atrium → Mitral valve → Left ventricle → Aortic valve → Aorta → Systemic circulation B. Superior and Inferior vena cavas → Right atrium → Mitral valve → Right ventricle → Pulmonary valve → Pulmonary artery → Lungs → Pulmonary veins → Left atrium → Tricuspid valve → Left ventricle → Aortic valve → Aorta → Systemic circulation C. Superior and Inferior vena cavas → Right atrium → Tricuspid valve → Right ventricle → Pulmonary valve → Pulmonary veins→ Lungs → Pulmonary artery → Left atrium → Mitral valve → Left ventricle → Aortic valve → Aorta → Systemic circulation D. Superior and Inferior vena cavas → Right atrium → Tricuspid valve → Right ventricle → Aortic valve → Pulmonary veins→ Lungs → Pulmonary artery → Left atrium → Mitral valve → Left ventricle → Pulmonary valve → Aorta → Systemic circulation Submit Answer

Explanation Choice A is correct. This is the proper blood flow through a healthy heart with normal anatomy. The superior and inferior vena cavas are the large veins that bring back deoxygenated blood from the body to the heart's right atrium. The blood enters the right atrium, passes through the tricuspid valve into the right ventricle, and is pumped into the lungs through the pulmonary artery. In pulmonary circulation, the deoxygenated blood drops off its carbon dioxide and waste products and picks up fresh oxygen to deliver to the body. It is now oxygenated. The blood returns to the left atrium through the pulmonary veins, passes through the mitral valve to enter the left ventricle, and is pumped out to the body through the aorta. Oxygenated blood is now in the systemic circulation, where it can deliver oxygen to all body tissues. Choice B is incorrect. In this sequence, the mitral and tricuspid valve locations are switched. Remember, the mitral valve is between the left atrium and ventricle, whereas the tricuspid valve is between the right atrium and ventricle. Choice C is incorrect. In this sequence, the pulmonary artery is switched with the pulmonary vein. The pulmonary artery carries deoxygenated blood away from the heart and to the lungs. It is the only artery in the body that has deoxygenated blood. The pulmonary vein brings oxygenated blood back from the lungs to the left atrium. Choice D is incorrect. In this sequence, the pulmonary and aortic valves are switched. The pulmonary valve is located at the opening of the pulmonary artery, whereas the aortic valve is located at the opening of the aorta. This should be easy to remember, as the pipes are named after the valve they open into. Last Updated - 18, Jan 2022

The nurse is caring for a client who was recently admitted to the cardiac floor for angina. This client states that their chest pain occurs at the same time every day at rest. The patient does not believe there are any precipitating factors. Which of the following types of angina is this patient most likely experiencing? A. Variant angina B. Stable angina C. Unstable angina D. Nonanginal pain Submit Answer

Explanation Choice A is correct. Variant angina, also known as Prinzmetal's angina, occurs at about the same time every day, usually at rest. Variant angina is treated with calcium channel blockers. Choice B is incorrect. Stable angina occurs after activity and is relieved by nitroglycerin tablets. Choice C is incorrect. Unstable angina is less predictable and may precipitate myocardial infarction. Choice D is incorrect. This type of discomfort does not describe nonanginal pain. NCSBN client need Topic: Physiological Integrity, physiological adaptation Last Updated - 12, Oct 2021

The patient with a right distal fibula fracture complains of pain and a tingling sensation in the right foot. Upon assessment, the nurse notes the right foot is cold to the touch with a weak dorsalis pedis pulse. Which potential complication should the nurse be most concerned about? A. Compartment syndrome B. Sepsis C. Peripheral neuropathy D. Pressure Injury Submit Answer

Explanation Choice A is correct. This patient is presenting with early signs/symptoms consistent with compartment syndrome. Later signs of compartment syndrome include paralysis and the absence of pulses in the affected extremity. If not caught and treated early, compartment syndrome can result in permanent muscle and nerve damage. Choice B is incorrect. There is no assessment data that supports a diagnosis of sepsis. Localized pain is expected in fracture patients, but would not be indicative of sepsis without additional symptoms of infection. Choice C is incorrect. Patients with nerve damage/peripheral neuropathy would experience symptoms such as pain and tingling. Still, the other assessment data of cold skin temperature and weak pulse would not support this diagnosis. Peripheral neuropathy would not be a more significant concern than compartment syndrome. Choice D is incorrect. This patient would be at risk of developing a pressure injury due to injury and immobility, but this potential complication would not be a more significant concern than compartment syndrome. NCSBN Client Need Topic: Adult Health - Musculoskeletal, Subtopic: System-specific assessments, medical emergencies, pathophysiology Additional Info Last Updated - 14, Jan 2022

The nurse prepares a client for a scheduled percutaneous coronary intervention (PCI). Which client statement should be reported to the primary healthcare provider (PHCP)? A. "I took my metformin this morning." B. "I get anxious when I am in closed spaces." C. "I am allergic to shellfish." D. "I may feel a warm sensation during the procedure." Submit Answer

Explanation Choice A is correct. This procedure involves intravenous (IV) contrast and a small chance of acute kidney injury may occur when IV contrast is given within 48 hours of metformin. Thus, the PHCP needs to be notified. Exposure to metformin prior to this procedure is not a contraindication but requires IV fluids to decrease the negative effects on the kidneys. Choices B, C, and D are incorrect. This procedure utilizes moderate sedation and is not done in a closed space. Thus, claustrophobia is not a concern. IV contrast is used during this procedure to define the coronary arteries. Contrary to a propagated myth, shellfish allergy has no relationship with IV contrast. IV contrast does cause an individual to experience a metallic taste and a warm sensation. This is an expected finding that does not require reporting to the PHCP. Additional Info Percutaneous coronary intervention is performed in the cardiac catheterization laboratory and combines clot retrieval, coronary angioplasty, and stent placement. Under fluoroscopic guidance, the cardiologist performs initial coronary angiography, inserting an arterial sheath and advancing a catheter retrograde through the aorta. Here the physician may determine which arteries are narrowed and require intervention. Intervention may come in the form of angioplasty with or without stenting. Last Updated - 17, Nov 2022

The nurse observes the following tracing on the telemetry monitor. The nurse should take which initial action? See the image below. A. Assess the client's level of consciousness B. Prepare the client for immediate defibrillation C. Administer a dose of intravenous epinephrine D. Evaluate the client's cardiac lead placement Submit Answer

Explanation Choice A is correct. This tracing depicts ventricular fibrillation. This rhythm is highly concerning because it can be fatal. Because the nurse has just seen this tracing on the telemetry monitor, the first action the nurse should take is to assess the client. Artifact may be confused for ventricular fibrillation, therefore the nurse should always assess the client first and not the monitor. Choices B, C, and D are incorrect. All of these actions may be appropriate for a client experiencing ventricular fibrillation. However, none of these assess the client's level of consciousness, which is the initial step in the basic life support algorithm. The client with ventricular fibrillation requires immediate defibrillation as the priority treatment if the client is unconscious and has no pulse. Additional Info Last Updated - 19, Jun 2022

The nurse is caring for a client with the following tracing on the electrocardiogram. When reviewing the client's medical history, which condition could be causing this dysrhythmia? See the image below. A. Graves' disease B. Increased intracranial pressure C. Severe hypothermia D. Myxedema coma Submit Answer

Explanation Choice A is correct. This tracing reflects sinus tachycardia (ST). ST can be caused by an array of conditions such as dehydration, hypo- and hyperglycemia, stress, anxiety, and thyroid conditions such as hyperthyroidism. Graves' disease is the most common cause of hyperthyroidism, and this increased metabolic and sympathetic activity would cause tachycardia. Choices B, C, and D are incorrect. Increased intracranial pressure would manifest with bradycardia. Hypothermia causes a slowing of metabolic and sympathetic activity; thus, bradycardia is a feature of this condition. Myxedema coma is an endocrine emergency marked by severe hypothyroidism. The hallmark of severe hypothyroidism is life-threatening bradycardia. Additional Info Sinus tachycardia is regular, with a p-Wave present. The rate of sinus tachycardia is over 100 beats per minute. Treatment of ST is the underlying cause; if the client is hypovolemic, fluids need to be replaced. ST may be worrisome as it is one of the earliest manifestations of shock. Last Updated - 26, Jun 2022

You are attending to a male client who is postoperative day one following mitral valve replacement. He is getting ready to ambulate for the first time. His heart rate is 102 beats/minute and the stroke volume based on the echocardiogram is 72 mL. Which of the following represents his cardiac output (CO)? A. 7.344 L/min B. 30 L/min C. 55% D. 73.444 mL/min Submit Answer

Explanation Choice A is correct. To answer this question correctly, you must know the right formula for Cardiac Output (CO). CO = Heart Rate (HR) x Stroke Volume (SV). Heart rate is measured in beats per minute and stroke volume is measured in milliliters (mL). The HR is simply the number of times per minute that the heart beats, whereas the SV is the mL of blood that the heart pumps out with each contraction. By multiplying the two together, you get how many mL of blood the heart is pumping out each minute. This is the cardiac output. Cardiac output is usually reported in liters/min; the average is about 5 L/min but varies greatly depending on the size of the patient. A decreased cardiac output (low-output failure) is seen in congestive heart failure. A high cardiac output state refers to resting cardiac output more significant than 8 L/min. An increased cardiac output (high-output failure) may be seen in hyperthyroidism, thiamine deficiency, and severe uncorrected anemia. For this problem: Cardiac Output (CO) = 102 beats per minute (HR) x 72 mL (SV) = 7,344 mL/min or 7.344 L/min. Choices B, C, and D are incorrect. Choice C appears to represent an ejection fraction, which can be obtained from an echocardiogram. Cardiac output should not be confused with ejection fraction. Ejection fraction (EF) is a "fraction" and is often reported as a percentage (%). Left ventricular ejection fraction is the amount of blood pumped out of the left ventricle during asystole. An ejection fraction of 55 percent means that 55 percent of the total amount of blood in the left ventricle is pumped out with each heartbeat. The normal EF is 55% to 70%. Ejection fraction is decreased in systolic heart failure, whereas it remains normal in diastolic heart failure. NCSBN Client Need Topic: Physiological Integrity; Subtopic: Basic care, comfort; cardiac Additional Info Source : Archer Review Last Updated - 18, Dec 2021

A patient presents to the emergency department with a dissecting aortic aneurysm. The patient needs immediate surgery to save his life. He is unconscious and there is no family contact information on file. Which action is appropriate for obtaining informed consent for the surgery? A. There is no need for obtained consent, send the client to surgery. B. Call the hospital lawyer. C. Search for people who may know the patient and can provide informed consent. D. Notify the nursing supervisor on-call and request her permission to waive informed consent. Submit Answer

Explanation Choice A is correct. When emergency surgery is needed, delaying the surgery to obtain informed consent may result in the patient's morbidity or death. In such urgent cases, informed consent is not necessary. It is most appropriate to begin the surgery to save the patient's life. Choice B is incorrect. This is not appropriate. The hospital lawyer does not need to be called. Choice C is incorrect. This is inappropriate and not necessary. In an emergency, it is both legally and ethically appropriate to start the operation without informed consent. Choice D is incorrect. The nursing supervisor on-call does not need to be notified, nor does she have the authority to waive the informed consent. Informed consent is not necessary due to the rationale in answer choice A. NCSBN Client Need: Topic: Psychosocial Integrity; Subtopic: Fundamentals of care; Ethical & Legal Issues Last Updated - 14, Nov

The nurse has just finished assisting the physician in applying a fiberglass cast to a client with a severe ankle sprain. The client inquires about the length of time to wait to walk on the cast. How much time will the nurse instruct the client to wait until walking again? A. 8 hours B. Half an hour C. After 24 hours D. After 48 hours Submit Answer

Explanation Choice B is correct. A fiberglass cast fully dries within 30 minutes (half an hour) of application. The client can walk (bear weight) on it if allowed by the physician after 30 minutes. Choices A, C, and D are incorrect. The client may bear weight (if permitted by the physician) on the cast once it is fully dry. A synthetic (fiberglass) cast fully dries after 30 minutes. A plaster cast takes 2-3 days to completely dry. Last Updated - 04, Jan 2022

The nurse is assessing a client with congestive heart failure. Which physical assessment finding should the nurse expect? A. Intermittent claudication B. S3 gallop C. Venous stasis ulcers D. Widened pulse pressure Submit Answer

Explanation Choice B is correct. An S3 gallop is an expected finding in heart failure. This is often an early manifestation of heart failure; it and this sound are best auscultated at the apex of the heart. Choices A, C, and D are incorrect. Intermittent claudication is a clinical feature of peripheral arterial disease. This causes a client pain as they ambulate and is relieved with rest. The pain may radiate to the ankle or buttock. Venous stasis ulcers are associated with long-term venous insufficiency; ulcer formed due to edema or minor injury to the limb; typically occurs over the malleolus. A widened pulse pressure (difference between the systolic and diastolic) is a feature of increased intracranial pressure, not heart failure. Additional Info A third heart sound, S3 gallop, is an early diastolic filling sound indicating an increased left ventricular pressure. This sound is often the first sign of HF. This sound is best auscultated over the apex of the heart. If this sound is auscultated during pregnancy, this is a normal finding. Last Updated - 31, Oct 2022

A post-coronary artery bypass graft patient developed a fever of 38.8° C. The nurse notifies the physician of the elevated temperature because: A. The elevated temperature may lead to profuse sweating. B. It may increase cardiac output. C. It is a sign of cerebral edema. D. It is indicative of hemorrhage. Submit Answer

Explanation Choice B is correct. An increase in temperature leads to increased metabolism and cardiac workload. Choice A is incorrect. Although there can be diaphoresis during an increase in temperature, it is not going to be a reason to call the physician. Choice C is incorrect. Fever is not an early sign of cerebral edema. Choice D is incorrect. When there is significant blood loss, there is no increase in temperature. Instead, there will be a decrease in temperature. Last Updated - 15, Feb 2022

The nurse is caring for a client who appears to be developing heart failure (HF). Which of the following laboratory tests would the nurse expect the primary health care provider (PHCP) to prescribe to confirm the diagnosis? A. Basic metabolic panel (BMP) B. B-type natriuretic peptide (BNP) C. Lipid profile D. Troponin Submit Answer

Explanation Choice B is correct. B-type natriuretic peptide (BNP) is a commonly ordered test for clients who may have heart failure. Elevations indicate worsening of heart failure as it is indicative of fluid retention. Choices A and C are incorrect. BMP and lipid profile tests are incorrect to confirm the diagnosis. Choice D is incorrect. A troponin laboratory test would be prescribed for acute coronary syndrome (ACS). Additional Info Last Updated - 31, Oct 2022

A 68-year-old woman arrives at the emergency department after feeling dizzy. After assessing the patient, the nurse notices hypotension, muffled heart tones, and jugular venous distention. What does the nurse suspect that this patient has? What is this triad called? A. Pericarditis; Cushing's triad B. Pericardial tamponade; Beck's triad C. Increased ICP; Cushing's triad D. Pleural effusion; Beck's triad Submit Answer

Explanation Choice B is correct. Beck's triad is a symptom triad that indicates pericardial tamponade. Hypotension occurs because the patient is actively losing blood into the pericardial space. This sac can hold as little as 150 mL to 1,000 mL and impedes cardiac output. Jugular vein distention (JVD) occurs because the heart is compressed, which leads to delayed venous return. Blood pools in the veins and this can be assessed as jugular vein distention. Choice A is incorrect. Cushing's triad is associated with increased ICP and is characterized by irregular respirations, widened pulse pressure, and bradycardia. Choice C is incorrect. Increased ICP and Cushing's triad is not what this patient is suffering from. Choice D is incorrect. This patient is experiencing symptoms that are called Beck's triad. However, this triad is indicative of pericardial tamponade, not a pleural effusion. A pleural effusion is known as fluid in the pleural space of the lung. NCSBN Client Need Topic: Physiological Adaptation, Sub-topic: Pathophysiology, Cardiovascular System Last Updated - 31, Dec 2021

The nurse is watching the monitor of a patient wearing a continuous cardiac monitor when it begins to alarm and fails to display any QRS complexes. Which nursing intervention should the nurse do first? A. Press record on the electrocardiogram B. Check the patient's lead placement C. Call the code team D. Contact the health care provider Submit Answer

Explanation Choice B is correct. Before calling a code or contacting the physician, the nurse should ensure that the leads are correctly placed on the patient and have not been removed. Physically looking and assessing the patient as well as the associated equipment should be the first action when an abnormal rhythm is noticed on the cardiac monitor. Choice A is incorrect. The first action is to ensure proper lead placement. Choice C is incorrect. Calling a code is not appropriate until the nurse has confirmed the patient is experiencing asystole. Choice D is incorrect. Contacting the care provider should not be completed until the nurse is sure that the patient's leads are working correctly. NCSBN client need Topic: Physiological Integrity, physiological adaptation Last Updated - 09, Jun 20

A postpartum client is preparing to be discharged home with her full-term newborn. Prior to discharge, the client verbalizes, "I really should not get pregnant again in the next three years so I can finish college." History reveals that she smokes a pack of cigarettes a day. Which method of contraception would be the most appropriate for this client? A. Medroxyprogesterone acetate injectable suspension B. Condoms and spermicidal contraceptive foam C. Natural family planning D. Oral contraceptives Submit Answer

Explanation Choice B is correct. Combining condoms and spermicidal contraceptive foam is highly effective in preventing pregnancy. In addition to being easily accessible, this method is also relatively inexpensive (often free if received from a local public health department). This combination is the most appropriate contraceptive method for this client. Choice A is incorrect. A hormonal contraception method such as medroxyprogesterone acetate injections increases the risk of clotting and strokes in women who smoke ten or more cigarettes daily. Therefore, following a review of the client's medical record and risk factors, this specific contraception method is not the best option for this client. Choice C is incorrect. Natural family planning involves intricate planning and timing of sexual contact around the menstrual cycle and signs of ovulation. Though this can be an effective method of birth control, it entails motivation, maturity, and strict attention to detail. Choice D is incorrect. The use of oral contraceptives would not be the ideal choice for this client. The use of oral contraceptives is contraindicated in women who are < 21 days postpartum or women who smoke > 15 cigarettes/day. Therefore, oral contraceptives are not the ideal choice for this client. Learning Objective When asked by this client for the most appropriate method of contraception, following a review of the client's risk factors, identify condoms and spermicidal contraceptive foam as the ideal method for this client. Additional Info The incidence of deep venous thrombosis and thromboembolism (e.g., pulmonary embolism) increases as the estrogen dose increases. Pregnancy rates tend to increase during the first year of use and decrease in subsequent years as users become more familiar with the chosen contraceptive method. In the United States, 12% of women who use contraception use intrauterine devices (IUDs). Last Updated - 22, Oct 2022

The nurse is teaching a group of clients about varicose veins and home care management. Which of the following should the nurse include in the teaching session? A. When you are sitting, keep your legs lower than your heart B. Wear compression stockings during the day C. Participate in activities that have you stand for long periods D. Take a low-dose aspirin to prevent the development of new varicose veins Submit Answer

Explanation Choice B is correct. Compression stockings/hose are effective because the external pressure promotes venous return. Compression hose combined with frequent position changes, daily walks, frequent position changes, and keeping the legs elevated to facilitate venous return is recommended. Choices A, C, and D are incorrect. These measures are not recommended when a client is managing varicose veins. Keeping the legs dependent is recommended for arterial insufficiency, where legs elevated is recommended to promote venous return. Standing in positions for a long period of time is not recommended. The nurse should advise the client to change positions frequently and engage in daily walks. Finally, aspirin may help with the pain associated with vaircose veins, but they do not prevent the formation of varicose veins. Additional Info Varicose veins can be managed by recommending that the client ✓ Wear compression hose/stockings ✓ Keep the legs elevated to promote venous return ✓ Engage in frequent position changes of the legs ✓ Daily walks are recommended; high impact exercise may contribute to the development of varicose veins Last Updated - 13, Jan 2023

The nurse cares for a client with suspected congestive heart failure (CHF). Which of the following laboratory tests would the nurse expect the primary health care provider (PHCP) to prescribe to confirm the diagnosis? A. Basic metabolic panel (BMP) B. B-type natriuretic peptide (BNP) C. Complete Metabolic Profile (CMP) D. C-Reactive Protein (CRP) Submit Answer

Explanation Choice B is correct. Congestive Heart Failure (CHF) may be confirmed by an elevation of the B-type natriuretic peptide (BNP). This peptide is elevated when it is cleaved from the ventricle wall because of increased ventricular filling pressures. Choices A, C, and D are incorrect. CHF is a condition that may lead to consequential fluid retention and decreased cardiac output. BMP, CMP, and CRP are not tests indicated for CHF. C-Reactive Protein may be used to detect atherosclerosis but not utilized in diagnosing heart failure. Additional Info Testing for CHF may consist of an electrocardiogram (ECG), echocardiogram, and laboratory work such as a BNP. When consistent with a diagnosis for CHF, the ECG may show left ventricular abnormalities. The echocardiogram may show ventricular dysfunction with reduced ejection fraction. A diagnosis of CHF is made when the ejection fraction is less than 40%. Last Updated - 31, Oct 202

The nurse is caring for a patient with a myocardial infarction who is experiencing tachycardia and coughs up frothy, pink-tinged sputum. Which finding would the nurse expect upon auscultation of lung sounds? A. Wheezing B. Crackles C. Rhonchi D. Diminished sounds Submit Answer

Explanation Choice B is correct. Crackles are common in cases of pulmonary edema related to myocardial infarction. Choice A is incorrect. Wheezing is common in cases of inflammation and narrowed airways. Choice C is incorrect. Rhonchi occurs in bronchitis and pneumonia. Choice D is incorrect. Diminished breath sounds are usually heard with COPD or pneumonia. NCSBN client need Topic: Physiological integrity, reduction of risk potential Last Updated - 04, Feb 2022

The nurse assists the code team with an unresponsive and pulseless client. Which intervention does the nurse prepare for based on the electrocardiogram (ECG) tracing? See the image below. A. Prepare an infusion of sodium bicarbonate B. Administer epinephrine C. Defibrillation D. Cardioversion Submit Answer

Explanation Choice B is correct. Epinephrine is necessary as this arrhythmia reflects asystole. Asystole (also known as ventricular standstill) requires an aggressive treatment consisting of high-quality cardiopulmonary resuscitation (CPR) and intravenous (IV) epinephrine. Epinephrine is necessary as this medication assists with restoring vascular tone. Choices A, C, and D are incorrect. A sodium bicarbonate infusion is not the essential treatment for asystole. While severe acidosis may contribute to life-threatening dysrhythmias, this is not an essential treatment. Defibrillation is the priority treatment in dysrhythmias such as ventricular fibrillation but is not helpful in asystole, considering no underlying rhythm is evident to shock. Cardioversion is an effective treatment for dysrhythmias such as atrial fibrillation and ventricular tachycardia (with a pulse). Additional Info A client with ventricular asystole has no pulse, respirations, or blood pressure. In some cases, the sinoatrial (SA) node may continue to fire and depolarize the atria, with only P waves seen on the ECG. However, the sinus impulses do not conduct to the ventricles, and QRS complexes remain absent. In most cases, the entire conduction system is electrically silent, with no P waves seen on the ECG. Essential treatment for ventricular asystole is high-quality CPR and intravenous (IV) push epinephrine. Last Updated - 28, Jun 2022

A hypertensive client was prescribed antihypertensive medication. The client tells a clinic nurse that she prefers to take an herbal supplement to help lower her blood pressure. Which is the most appropriate response from the nurse? A. Tell the client that herbal supplements are unsafe and should be avoided B. Encourage the client to discuss the use of herbal supplements with her attending health care provider (HCP) C. Teach the client how to take her blood pressure and ask her to monitor it every fifteen minutes D. Tell the client that if she takes the herbal supplement, it will require the nurses to monitor her blood pressure closely Submit Answer

Explanation Choice B is correct. Herbal supplements are not regulated in the same manner as traditional pharmacy-dispensed medications. Although the use of some herbal supplements may provide some beneficial effects, not all herbal supplements are safe for use. Clients on conventional pharmaceutical therapy (i.e., hypertensive medications) should be discouraged from using herbal supplements, especially those with similar pharmacological effects, as the combination may produce an excessive reaction of unknown interaction effects. The nurse should advise the client to discuss all herbal supplements (regardless of whether they address hypertension or not) with her attending HCP. Choice A is incorrect. Some herbal supplements may benefit some clients; therefore, it would be wrong for the nurse to make a blanket statement such as "herbal supplements are unsafe and should be avoided" to this client. Choice C is incorrect. Although the nurse should teach the client to take and monitor her blood pressure, doing this every 15 minutes is much too frequently. Choice D is incorrect. While the client should be informed that the use of an herbal supplement would prompt the need for the nurses to monitor her blood pressure closely, this would not be the priority response out of the choices provided. Here, the question asks for the most appropriate response from the nurse. Learning Objective Identify the need to instruct a client to speak with her health care provider (HCP) following the client's disclosure of her preference to take an herbal supplement to help control hypertension. Additional Info Because herbs are regarded as food products, they are not subject to the same scrutiny and regulation as traditional medications. As a result, manufacturers are exempt from premarket safety and efficacy testing before the release of an herbal product and from any post-marketing surveillance. Although herbal remedies are perceived as being natural and therefore safe, many have adverse effects that can sometimes produce life-threatening consequences. Common herbal remedies that produce adverse effects on the cardiovascular system: include St. John's wort, motherwort, ginseng, gingko biloba, garlic, grapefruit juice, hawthorn, saw palmetto, danshen, echinacea, tetrandrine, aconite, yohimbine, gynura, licorice, and black cohosh. Last Updated - 12, Sep 2022

The client with an implantable cardioverter defibrillator (ICD) is at the outpatient department. He is concerned about a shock that his friend felt when they were shaking hands. The nurse can discuss which of the following in response to the patient's concern? A. His friend should have an ECG taken to check if his heart rhythm was affected in anyway. B. He shook hands with his friend at the exact same time the ICD delivered a shock to restore his rhythm, and that he need not worry. C. The shock was due to the magnetic field the device emits. D. He should get urgently checked by the physician. Submit Answer

Explanation Choice B is correct. If the ICD administers shock, others in physical contact with the patient may feel it but will not be harmed. A shock from the ICD indicates that it's effectively treating the rhythm disorder. Choice A is incorrect. If the ICD administers shock, others in physical contact with the patient may feel it but will not be harmed. Choice C is incorrect. The ICD does not produce any magnetic field. Choice D is incorrect. The shock felt by the patient's friend is an expected outcome of a functional ICD that's treating the patient's rhythm. The question does not mention any clinical symptoms such as chest pain following the shock. This does not necessitate an immediate checkup by a physician. If the patient had just one shock in 24 hours and feels fine after the shock; no immediate intervention is necessary. However, if the patient reports chest pain/ chest pressure/ shortness of breath following an ICD shock or if two or more shocks are experienced within a 24 hour period, the patient should seek medical attention immediately. Last Updated - 11, Aug 2022

How should the nurse assess for the presence of thrombophlebitis in a patient who reports having pain in the left lower leg? A. By palpating the skin over the tibia and fibula B. By documenting daily calf circumference measurements C. By recording vital signs obtained four times a day D. By noting difficulty with ambulation Submit Answer

Explanation Choice B is correct. Inflammation from thrombophlebitis increases the size of the affected extremity and can be assessed by measuring circumference regularly. Thrombophlebitis is an inflammation of a vein associated with thrombus formation. Thrombophlebitis from venous stasis is most commonly seen in the legs of postoperative patients. Manifestations of thrombophlebitis are pain and cramping in the calf or thigh of the involved extremity, redness and swelling in the affected area, elevated temperature, and an increase in the diameter of the involved extremity. Each shift, nurses should assess the legs for swelling and tenderness, measure bilateral calf or thigh circumference, and determine if the patient experiences any chest pain or dyspnea. The patient should be instructed not to massage the legs. Choices A, C, and D are incorrect. These options are not the correct way to assess for the presence of thrombophlebitis. NCSBN Client Need Topic: Safe and Effective Care Environment, Subtopic: Safety and Infection Control; Thrombophlebitis Last Updated - 14, Feb 2022

The nurse is performing a physical assessment on a client with infective endocarditis (IE). The nurse observes flat, reddened non-tender maculae on the hands and feet. The nurse understands that these are A. Heberden's nodes. B. Janeway lesions. C. tophi. D. Bouchard's nodes. Submit Answer

Explanation Choice B is correct. Janeway lesions are common with infective endocarditis (IE). The cause of these findings are the cause of the lesions are septic microemboli from the valvular lesion. These macules are not painful and are typically located on the palms, soles, and plantar surfaces of the toes. Choices A, C, and D are incorrect. Heberden's and Bouchard's nodes are associated with arthritis. Heberden nodes are bony nodules at the distal interphalangeal [DIP] joints. Bouchard's nodes are bony nodules at the proximal interphalangeal [PIP] joints. Tophi are dermal and subcutaneous deposits of urate crystals associated with gout. Additional Info Infective endocarditis occurs primarily in patients with injection drug use (IDU) and those who have had valve replacements, have experienced systemic alterations in immunity, or have structural cardiac defects. This is a condition caused by the invasion of bacteria which enter the client through contaminated needles, oral cavity following dental procedures, and/or skin abscesses. Manifestations of IE include fever, anorexia, weight loss, cardiac murmur, petechiae, Janeway lesions, and splinter hemorrhages are commonly observed with IE. Last Updated - 16, Jul 2022

The nurse is caring for a client with the following clinical data. The nurse should expect the primary healthcare provider (PHCP) to prescribe what medication? See the exhibit. View Exhibit A. Enalapril B. Labetalol C. Amiodarone D. Nitroglycerin Submit Answer

Explanation Choice B is correct. Labetalol is an alpha- and beta-adrenergic blocking agent used to treat a hypertensive emergency. Considering that this client is both hypertensive and tachycardic, labetalol would be a good choice. Choices A, C, and D are incorrect. Enalapril is an ACE inhibitor that may be given intravenously. However, this medication would have no effect on the client's tachycardia. Thus, this would be an inappropriate recommendation. Amiodarone is an umbrella drug used to manage atrial and ventricular dysrhythmias. This medication is not indicated for a hypertensive emergency. While utilized in treating a hypertensive emergency, nitroglycerin would not be efficacious for the client's tachycardia. It would make the tachycardia worse as this drug causes reflex tachycardia as the blood pressure is decreased. Additional Info A hypertensive emergency (or sometimes termed hypertensive crisis) is defined as a blood pressure of 180/120 mmHg or greater. This blood pressure may cause end-organ damage and symptoms such as flushing, headache, dizziness, angina, and/or dyspnea. Nursing care aims to establish prompt intravenous (IV) access, position the client in the semi-Fowler's position, initiate continuous cardiac monitoring, and monitor the blood pressure frequently. The blood pressure should be lowered gradually, with the mean arterial pressure lowered by 10-20 percent within the first hour. Last Updated - 03, Dec 2022

Which of the following lipid levels are out of range and should be reported to the physician? A. Triglycerides: 75 mg/dL B. Total cholesterol: 6.5 mmol/L C. High-density lipoprotein (HDL): 60 mg/dL D. Low-density Lipoprotein (LDL): 95 mg/dL Submit Answer

Explanation Choice B is correct. Lipid profile helps physicians determine the patient's risk of developing heart disease. It is recommended that individuals have a lipid profile done at least every five years as part of a regular medical exam. 6.5 mmol/L exceeds the "high normal" total cholesterol level. The average total cholesterol level is 3.5 to 5.0 mmol/L. In milligrams, total cholesterol of 200 milligrams per deciliter (mg/dL) or less is considered desirable for adults Choices A, C, and D are incorrect. The normal lipid levels for these tests include: Triglycerides: 50-150 mg/dL High-density lipoprotein (HDL): 40-80 mg/dL Low-density lipoprotein (LDL): 85-125 mg/dL NCSBN Client Need Topic: Physiological Integrity; Subtopic: Reduction of Risk Potential Last Updated - 19, Dec 2021

The nurse in the post-anesthetic care unit has just received a patient from the OR that has undergone a coronary artery bypass graft (CABG). The nurse formulates a nursing diagnosis of "decreased cardiac output related to alterations in preload/afterload/contractility/heart rate." What nursing interventions should be implemented in the nursing care plan based on the formulated nursing diagnosis? A. Monitor the patient's arterial blood gas constantly. B. Monitor the patient's weight daily and calculate the change. C. Administer prescribed opioids. D. Monitor mediastinal chest tubes for hourly output. Submit Answer

Explanation Choice B is correct. Monitoring the daily weight of the client and noting any changes gives the nurse a picture of the client's fluid volume status, which is influenced by the patient's cardiac output. Choice A is incorrect. Monitoring the patient's arterial blood gas (ABG) is also needed in the care of a patient post-CABG to detect hypoxemia or acidosis, which may require modification in ventilation parameters. However, it is not directly related to the nursing diagnosis of decreased cardiac output; it is an intervention to address the nursing diagnosis of impaired gas exchange. Choice C is incorrect. Opioids provide pain relief to the client after surgery and should be administered by the nurse promptly to manage pain. However, opioid administration is not a nursing intervention related to the care of a patient with a nursing diagnosis of decreased cardiac output. Choice D is incorrect. Hourly monitoring of mediastinal chest tubes assesses the patient's rate of blood loss post-surgery. This is a nursing intervention related to risk for illness related to inadequate hemostasis, not decreased cardiac output. Last Updated - 14, Jan 2022

Which of the following statements best describes the cardiovascular system? A. It has a heart with six chambers, strong vessels, and valves. B. It is a double-pump circulating blood out to the lungs and the body. C. It includes concepts of precontractility, postcontractility, and load. D. It functions with a conduction system and starts in the ventricles. Submit Answer

Explanation Choice B is correct. The heart is a double pump with four chambers, four valves, and a conduction system with a pacemaker originating in the atrium. Choice A, C, and D are incorrect. The heart has four chambers, not six. These chambers include two atria (right and left) and two ventricles (right and left). Blood from the entire body returns to the heart's right atrium through the superior and inferior vena cavaes. Blood circulates through the right atrium, then to the right ventricle, gets oxygenated in the lungs, moves on to the left atrium, then the left ventricle, and is pumped back to the systemic circulation via the aorta. The conduction system of the heart begins in the right atrium, not the ventricle. The heart's conduction system includes pacemaker cells (SA node, AV node, Bundle of His, bundle branches, and Purkinje fibers) plus contractile cells. The sinoatrial node (SA node), located in the right atrium, is the pacemaker that sets the heart rate and is the starting point of the conduction system. The effectiveness of the pumping action of the heart is described in concepts of preload, afterload, and contractility. Preload (end-diastolic volume) is the amount of initial stretching of the ventricles before the contraction (systole) begins. Preload is determined by the venous return to the heart and is directly related to ventricular filling. Afterload refers to the resistance/load against which the left ventricle pumps out the blood. Afterload is directly determined by aortic pressure (systemic vascular resistance, SVR). Finally, contractility (inotropy) refers to the innate ability of heart muscle to contract at a given afterload and preload. Preload, afterload, and contractility determine the stroke volume and ejection fraction. Learning Objective Understand the normal anatomy and physiology of the cardiovascular system. Additional Info Understanding the concepts of preload, afterload, stroke volume, and peripheral vascular resistance is essential to understand the pathophysiology of heart failure and the rationale for using certain medications in heart failure. Several drugs are used in heart failure and work by different methods. Diuretics and nitrates reduce the preload. Anti-hypertensive medications such as angiotensin-converting enzyme inhibitors (ACEI) and angiotensin receptor blockers (ARBs) work by reducing both the preload and afterload. ACEI and ARBs are the drugs of choice in the long-term management of congestive heart failure. Positive inotropic drugs such as digoxin, dopamine, dobutamine, and milrinone directly increase contractility and are often used in acute heart failure. Last Updated - 20, Jan 2022

The nurse is caring for a client who has just been diagnosed with acute pericarditis. Which of the following medications should the nurse anticipate the primary health care provider (PHCP) will prescribe? A. Isoniazid B. Colchicine C. Allopurinol D. Warfarin Submit Answer

Explanation Choice B is correct. The initial treatment for acute pericarditis includes NSAIDs or colchicine. Colchicine may be combined with an NSAID in the treatment of pericarditis. Choices A, C, and D are incorrect. Isoniazid is indicated for pulmonary tuberculosis, allopurinol is indicated for gout, and warfarin is an anticoagulant used to prevent thrombosis. NCLEX Category: Pharmacological and Parenteral Therapies Activity Statement: Expected Actions/Outcomes Question type: Knowledge/comprehension Additional Info Acute pericarditis can be managed with NSAIDs, colchicine, or glucocorticoids. Major side effects of colchicine include gastrointestinal upset, which may be mitigated by taking this medication with food. Colchicine may be added to NSAID therapy adjunctively to maximize treatment response. Last Updated - 28, Apr 20

The nurse in the emergency department is taking care of a patient diagnosed with left ventricular failure. The patient presents with fatigue, muscular weakness, and dyspnea. The patient is seen coughing and sitting in a "three-point position". The nurse understands that manifestations of left-sided heart failure present themselves as respiratory problems because: A. There is venous congestion in the liver. B. There is hypoperfusion of tissue cells. C. There is pulmonary congestion. D. Despite the normal cardiac output, the heart is still not able to meet the accelerated demands of the body. Submit Answer

Explanation Choice C is correct. Due to the inability of the left ventricle to pump blood, there is an accumulation of blood behind it, leading to congestion in the pulmonary veins down to the lungs. Choice A is incorrect. Venous congestion in the liver occurs because of a decrease in the functioning of the right ventricle. Choice B is incorrect. Hypoperfusion of tissues is a consequence of most forms of heart failure. However, the manifestations of left heart failure occur because of pulmonary congestion. Choice D is incorrect. The heart being unable to meet the accelerated needs of the body despite its standard cardiac output is a description of high output heart failure. This occurs in sepsis, Paget's disease, beriberi, anemia, and other conditions. Additional Info Last Updated - 31, Oct 2022

An emergency department (ED) nurse establishes continuous cardiac monitoring for a client. The following tracing is observed on the monitor. The nurse should take which initial action? See the image below. A. Establish vascular access and request a prescription for atropine B. Assess the client's blood pressure and level of consciousness C. Obtain and review the client's current medications D. Document the findings and reassess the client in one hour Submit Answer

Explanation Choice B is correct. The nurse should prioritize assessing the client's vital signs and level of consciousness. This tracing reflects sinus bradycardia. While sinus bradycardia may be benign, if the client should experience unstable blood pressure or dizziness, the nurse will need to act by establishing vascular access and administering atropine. However, this is predicated on the client's overall stability, which can only be discerned by assessment. Choices A, C, and D are incorrect. These actions are plausible but do not prioritize assessing the client and their overall condition. Reviewing the client's current medications may determine the origin of the bradycardia but will not yield clues as to the client's current level of stability. Documentation should only occur once the nurse has determined that the client is stable. Additional Info Sinus bradycardia is a regular rhythm with a rate of less than sixty. Sinus bradycardia is only concerning if the client is symptomatic. Pathological causes of bradycardia include severe hypothyroidism, hypothermia, anorexia nervosa, and prolonged hypoxia. Last Updated - 11, Jan 2023

The patient that has just undergone cardiac surgery is recovering in the post-anesthetic care unit. The nurse notices that the patient's blood pressure is 88/52 mmHg and that his jugular veins are very prominent. The nurse auscultates his heart rate and cannot hear any heart sounds. The nurse immediately informs the physician on duty and prepares for which procedure: A. Thoracentesis B. Pericardiocentesis C. Arthrocentesis D. Paracentesis Submit Answer

Explanation Choice B is correct. The patient in the situation is experiencing a cardiac tamponade, manifested by hypotension, distended neck veins, and inaudible heart sounds (Beck's Triad). If cardiac tamponade is suspected, treatment involves pericardiocentesis to relieve the compression of the ventricles. Choice A is incorrect. Thoracentesis is the removal of fluid between the lungs and the chest wall (pleural space). The patient does not show any symptoms indicating fluid in the pleural space. Thus, it is unnecessary. Choice C is incorrect. Arthrocentesis is the aspiration of synovial fluid from the joints. The patient in the situation does not have any signs or symptoms relating to problems in his bones. Choice D is incorrect. Paracentesis is the removal of fluid from the abdominal cavity to relieve pressure from the abdomen. The patient is displaying symptoms that show compression in the ventricles of the heart. Therefore, paracentesis is not necessary. Last Updated - 08, Jan 2022

While performing cardiac auscultation on a client. The nurse notes a whooshing and blowing sound over the heart valves. The nurse knows that this sound can be identified as a: A. Pericardial friction rub B. Heart murmur C. Normal lub-dub sounds D. S3 Submit Answer

Explanation Choice B is correct. The whooshing or blowing sound sometimes heard upon cardiac auscultation is known as a heart murmur and may indicate valve incompetency. Choice A is incorrect. Pericardial friction sounds like a scratching sound caused by the conflict between the heart and the pericardium. Choice C is incorrect. Whooshing and blowing do not indicate normal lub-dub sounds. Choice D is incorrect. S3 is a third heart sound, sometimes referred to as a gallop. This gallop is not the same thing as a murmur. Last Updated - 05, Feb 2022

The nurse cares for a client with the below tracing on the electrocardiogram. The client is unresponsive and without a pulse. The nurse should take which priority action based on the tracing? A. Start cardiopulmonary resuscitation (CPR) B. Perform immediate defibrillation C. Initiate intravenous (IV) access D. Review the client's most recent electrolyte levels Submit Answer

Explanation Choice B is correct. This concerning tracing on the electrocardiogram is ventricular fibrillation. Ventricular fibrillation is electrical chaos in the ventricles that produces no cardiac output. The priority is to defibrillate the client immediately according to ACLS protocol. If a defibrillator is not readily available, high-quality CPR must be initiated and continued until the defibrillator arrives. Choices A, C, and D are incorrect. CPR is a critical intervention in the management of ventricular fibrillation. However, it is not a higher priority than defibrillation. Defibrillation has been proven to terminate this lethal arrhythmia and is essential to perform. IV access is essential to obtain to deliver medications such as epinephrine, but restoring normal cardiac output will not come through IV access. Ventricular fibrillation may be caused by various conditions, including severe electrolyte disturbances. However, this client is in distress, and assessment is not the priority. Additional Info Ventricular fibrillation causes the ventricles to merely quiver, consuming a tremendous amount of oxygen. There is no cardiac output or pulse and therefore no cerebral, myocardial, or systemic perfusion. This rhythm is rapidly fatal if not successfully ended within 3 to 5 minutes. The essential treatment of ventricular fibrillation is defibrillation, and CPR is merely a life-sustaining measure that should be used until a defibrillator arrives. Last Updated - 19, Jun 2022

A nurse is reviewing instructions with a client for the client's upcoming exercise electrocardiography (ECG). All of the following are appropriate statements by the nurse, except for which? A. "Please wear loose, comfortable clothing with non-slip athletic footwear." B. "You may have a light breakfast and a small cup of coffee before the exam." C. "We will monitor your blood pressure, heart rate, and rhythm before, during, and after the stress test." D. "You should not do any strenuous activity before your stress test." Submit Answer

Explanation Choice B is correct. This statement by the nurse is incorrect and, therefore, the correct answer to the question. Typically, clients scheduled to undergo an exercise electrocardiography (ECG) (i.e., exercise stress test) are instructed to remain NPO for 4 to 6 hours before the stress test. Additionally, caffeine intake by a client may cause an alteration in the procedure's findings. Choice A is incorrect. The client should be instructed to wear loose, comfortable clothing with non-slip supportive footwear. In addition to facilitating exercise, stress test technicians will require access to the client's chest wall and all four extremities numerous times throughout the examination. Loose clothing will assist in facilitating access to those areas. Choice C is incorrect. The client will have continuous monitoring of their heart rate and rhythm (i.e., electrocardiogram) during the examination. A blood pressure cuff will also be continuously worn by the client during the testing, with readings taken at specific intervals. Choice D is incorrect. The client should be instructed to be well rested for the examination, refraining from strenuous activities the day and evening before the testing. Learning Objective Recognize that caffeine intake can alter exercise stress test results and should therefore be avoided by a client prior to the procedure. Additional Info In exercise stress testing, the client walks on a conventional treadmill, following a specific protocol or exercise schedule, until the target heart rate is reached or symptoms occur. Last Updated - 19, Aug 2022

The nurse reviews a client's understanding of newly prescribed nitroglycerin sublingual tablets. Which of the statements, if made by the client, would require follow-up? A. "I will get a refill of my prescription every six months." B. "I will take one tablet every 2 minutes if chest pain occurs." C. "I will place my medication in a dark amber bottle." D. "I must not chew on the tablet when taking it." Submit Answer

Explanation Choice B is correct. This statement is incorrect and requires follow-up. When chest pain occurs, the client should take one tablet of nitroglycerin sublingually every five minutes for three doses. Taking the medication too frequently may result in severe hypotension. Choices A, C, and D are incorrect. These statements are factual and do not require follow-up. Nitroglycerin expires after six months and should be replaced. The client should seek emergency care if the chest pain does not improve after the first dose. Nitroglycerin tablets are not to be swallowed or chewed but kept under the tongue. Finally, nitroglycerin is photosensitive and should be kept in the dark amber bottle. Additional Info Nitroglycerin is a potent vasodilator (it decreases preload and afterload). It is indicated in angina. It is given in a variety of preparations, including sublingual, translingual, and topical. Dosing for sublingual nitroglycerin is one tablet under the tongue every five minutes (as long as the chest pain is persisting). The maximum tablet (or sprays) is three. The client should be instructed that emergency care should be sought if the pain is not relieved after the first dose. Nitroglycerin expires after six months, and the client should be instructed to keep their supply current. Nitrates are contraindicated if the client is taking medications such as vardenafil, tadalafil, or sildenafil. The client should take the nitroglycerin in a sitting or laying down position because sudden movement changes may cause orthostatic hypotension. Headache is an expected side effect of this medication and may be treated with acetaminophen. Nitroglycerin cannot be applied to a client for 24 hours as the client will develop a tolerance. Blood pressure should be monitored closely. Last Updated - 31, Jul 2022

The nurse is caring for a client with the following tracing on the electrocardiogram (ECG). The nurse identifies this tracing as See the image below. A. sinus tachycardia. B. supraventicular tachycardia. C. ventricular tachycardia. D. atrial flutter. Submit Answer

Explanation Choice B is correct. This tracing reflects SVT, which is concerning because of the very high rate. The rate may be as high as 150 to 250 beats/min in adults. Characteristically, SVT does not have P-waves as they are buried in the T-waves. Choices A, C, and D are incorrect. Sinus tachycardia (ST) is excluded because ST has noticeable P-waves proceeding each QRS complex. Ventricular tachycardia (VT) is excluded because while this dysrhythmia may have a rate up to 180 beats/min, the QRS complex is noticeably widened. Atrial flutter is excluded because of its gross irregularity. Additional Info During SVT, P waves may not be visible, because the P waves are embedded in the preceding T wave. A client with SVT may be asymptomatic. If the client is symptomatic, they may exhibit manifestations such as palpitations, dizziness, dyspnea, and nervousness. Treatment includes vagal maneuvers. Vagal maneuvers include having the beardown, blowing through a straw, having the primary healthcare provider (PHCP) perform a carotid massage, and, if the client is an infant, applying an ice pack firmly to the infant's head. If these measures are ineffective, the nurse should prepare to administer the prescribed adenosine by rapid intravenous push (IVP) that is followed by a flush of 0.9% saline. When adenosine is administered, the emergency (code) cart should be nearby, and the nurse should always have additional personnel in the room. Last Updated - 27, Jun 2022

The nurse is caring for an infant with the below tracing on the electrocardiogram (ECG). The nurse should plan to take which initial action? See the image below. A. Instruct the infant to beardown. B. Prepare a bag filled with ice and water. C. Assess the infant's axillary temperature. D. Obtain the infant's carotid pulse. Submit Answer

Explanation Choice B is correct. This tracing reflects SVT, which is concerning because of the very high rate. The rate may be as high as 180 to 280 beats/min in infants. Characteristically, SVT does not have P-waves as they are buried in the T-waves. Preparing a bag filled with ice and water is essential because this may be applied to the face above the nose and mouth for 15 to 30 seconds. If that is not effective, another vagal maneuver would be pressing the infant's knees to the chest for 15-30 seconds. Choices A, C, and D are incorrect. Instructing an infant to beardown would be futile because the infant does not have the cognitive ability to follow this instruction. This is the instruction that could be provided to a child or adult. Assessing the infant's axillary temperature would not have clinical significance because of the necessity to break the SVT. While SVT may be caused by severe febrile illness, the infant's rhythm must be corrected. The pulse on an infant should be obtained via the brachial artery. Additional Info During SVT, P waves may not be visible, because the P waves are embedded in the preceding T wave. A client with SVT may be asymptomatic. If the client is symptomatic, they may exhibit manifestations such as palpitations, dizziness, dyspnea, and nervousness. Treatment includes vagal maneuvers. Vagal maneuvers include having the beardown, blowing through a straw, having the primary healthcare provider (PHCP) perform a carotid massage, and, if the client is an infant, applying a bag filled with ice and water to the face above the nose and mouth for 15 to 30 seconds. If that is not effective, another vagal maneuver would be pressing the infant's knees to the chest for 15-30 seconds. If these measures are ineffective, the nurse should prepare to administer the prescribed adenosine by rapid intravenous push (IVP) that is followed by a flush of 0.9% saline. When adenosine is administered, the emergency (code) cart should be nearby, and the nurse should always have additional personnel in the room. Last Updated - 27, Jun 2022

The nurse is caring for a client who has sustained a myocardial infarction. Which cardiac enzyme should the nurse expect to be elevated in response to myocardial injury? A. CPK-MB B. Troponin C. Creatinine kinase D. Myoglobin Submit Answer

Explanation Choice B is correct. Troponin is the most specific iso-enzyme when evaluating a client's myocardial infarction (MI). Troponin levels will elevate within 3-4 hours of myocardial infarction and remain elevated for three weeks. This means that troponin is the most specific cardiac biomarker for an MI and is the most reliable test to run if the client does not seek care for some time after their symptoms begin. Choices A, C, and D are incorrect. CPK-MB is not the most specific iso-enzyme to myocardial infarction. CPK-MB is a cardiac iso-enzyme, and levels of this iso-enzyme do become elevated with damage to the cardiac cells of the myocardium; it is just not the most specific. Levels of CPK-MB increase about 3 to 6 hours after there is damage to the heart muscle, and their levels peak in 24 hours. Creatine kinase is not the most specific iso-enzyme to myocardial infarction. It is an enzyme found in the heart, brain, skeletal muscle, and other tissues. Any time there is muscle damage, increased amounts of creatinine kinase are released into the blood. This means, however, that it is not a sensitive indicator for myocardial infarction because muscle damage of any kind will cause elevated levels. When running this lab after a myocardial infarction is suspected, negative results are a good thing to aid in ruling out an MI. Still, positive results are not specific, and further testing will need to be done. Myoglobin is not an iso-enzyme but a red protein containing heme, which carries and stores oxygen in muscle cells. Myoglobin levels increase after myocardial infarction because any muscle breakdown releases myoglobin into the bloodstream. This means, however, that it is not a sensitive indicator for myocardial infarction because muscle damage of any kind will cause elevated levels. When running this lab after a myocardial infarction is suspected, negative results are a good thing to aid in ruling out an MI. Still, positive results are not specific, and further testing will need to be done. Additional Info Troponin is a myocardial muscle protein released into the bloodstream with injury to myocardial muscle. Any rise in values indicates cardiac necrosis or acute MI. Specific markers of myocardial injury, troponins T and I, have a wide diagnostic time frame, making them useful for patients who present several hours after the onset of chest pain. Even low levels of troponin T are treated aggressively because of the increased risk for cardiovascular disease (CVD)

While auscultating a client's bowel sounds, the nurse notes a swooshing sound to the left of the umbilical area. What would be the nurse's priority action? A. Percuss over the area to assess for dullness B. Notify the primary healthcare provided (PHCP) C. Gently palpate the abdomen to assess for tenderness D. Ask the patient about recent bowel movements Submit Answer

Explanation Choice B is correct. Upon auscultation, the nurse should suspect this client is presenting with an abdominal aortic aneurysm (AAA) due to the bruit or swooshing sound. The nurse should immediately notify the patient's healthcare provider of this urgent situation. An AAA Rupture can occur spontaneously or with trauma. If the aneurysm bursts, it may cause life-threatening bleeding. The aneurysm should be assessed immediately to determine the need for surgical intervention. Choices A, C, and D are incorrect. If a bruit is heard upon auscultation of the abdominal aorta, the nurse should suspect an aneurysm and not perform percussion or palpation due to the risk of rupture. Asking the patient about bowel movements (choice D) would be appropriate for assessing the patient's gastrointestinal system but would not be the highest priority for a suspected aneurysm. Additional Info Assessment findings concerning an abdominal aortic aneurysm (AAA) include a bruit in the abdominal region. The PHCP should order a computed tomography (CT) scan to verify the presence of an AAA. If the client has had a rupture, manifestations include gnawing-like back (or flank) pain that may radiate to the groin or buttocks. Last Updated - 01, Nov 2022

The nurse in the medical-surgical unit is observing the telemetry monitor for assigned clients. Which client condition change requires immediate notification to the primary healthcare provider (PHCP)? A client A. with normal sinus rhythm (NSR), who has had three premature ventricular contractions (PVCs) in the past hour. B. recovering from cardiac catheterization and has had two brief runs of ventricular tachycardia (VT). C. with atrial fibrillation, whose heart rate decreased from 113 to 95 beats per minute. D. who has developed sinus tachycardia (ST) following the application of nitroglycerin paste. Submit Answer

Explanation Choice B is correct. Ventricular tachycardia (VT) is an ominous dysrhythmia that may portend the client clinically deteriorating. Following a cardiac catheterization, the client is at risk for an array of complications, including hemorrhage, cardiac dysrhythmias, and reinfarction. Although brief, these two runs of VT need to be reported to the PHCP immediately, so prescribed medications such as amiodarone may be given to prevent further ventricular irritability. Choices A, C, and D are incorrect. PVCs are generally benign and may occur with anxiety, central nervous stimulants, or electrolyte imbalances. The individual experiencing a PVC may experience a brief palpitation. If the PVCs continue to persist, the PHCP may order magnesium and potassium levels because low levels of either may trigger PVCs. Rate control is a goal for a client with atrial fibrillation. The client's rate has decreased within the normal range (60-100/minute) and does not require an immediate PHCP notification. Reflex sinus tachycardia is common with nitroglycerin because of the significant vasodilation triggers an increased heart rate to maintain appropriate cardiac output. Additional Info Ventricular tachycardia may present with or without a pulse. Thus, the nurse must assess the client before intervening. ✓ For a client with ventricular tachycardia (VT) and a pulse, the nurse should remain with the client and anticipate a prescription for intravenous (IV) amiodarone or lidocaine. ✓ Another treatment for VT with a pulse will be cardioversion if the prescribed medications are unavailable or ineffective. ✓ For a client with ventricular tachycardia and no pulse, the nurse should remain with the client and perform high-quality CPR. ✓ The priority treatment for pulseless ventricular tachycardia would be prompt defibrillation. Last Updated - 08, Jan 2023

Which of the following expected outcomes is appropriate for a client with heart disease who is complaining of chest pain? A. The client will be free of neuropathic pain related to angina. B. The client will be free of hyperalgesia pain related to angina. C. The client will be free of visceral pain related to angina. D. The client will be free of somatic pain related to angina. Submit Answer

Explanation Choice C is correct. "The client will be free of visceral pain related to angina" is an appropriate expected outcome for a client with heart disease who is complaining of chest pain. Chest pain is an example of visceral pain. Other cases of physical pain are cramping secondary to irritable bowel syndrome and labor pain. Choice A is incorrect. "The client will be free of neuropathic pain related to angina" is not an appropriate expected outcome for a client with heart disease who is complaining of chest pain. Chest pain is not neuropathic pain. Choice B is incorrect. "The client will be free of hyperalgesia pain related to angina" is not an appropriate expected outcome for a client with heart disease who is complaining of chest pain. Chest pain is not hyperalgesia. Choice D is incorrect. "The client will be free of somatic pain related to angina" is not an appropriate expected outcome for a client with heart disease who is complaining of chest pain. Chest pain is not bodily pain. Last Updated - 05, Feb 2022

The nurse is assessing a cardiac rhythm strip with the characteristics shown in the exhibit. The nurse should plan to document this rhythm as which of the following? See the exhibit. View Exhibit A. Ventricular fibrillation B. Complete (3rd degree) heart block C. Sinus tachycardia D. Sinus bradycardia Submit Answer

Explanation Choice C is correct. Based on the information provided, this rhythm is sinus tachycardia. The PR and QRS intervals, as well as the rhythm, are normal. The rate of 105 is high, suggesting tachycardia. Choices A, B, and D are incorrect. Ventricular fibrillation is irregular and has no discernible P or QRS waves. The rate of ventricular fibrillation is also having a rate ranging anywhere from 150 - 500. A complete heart block has a variable PR interval because it is completely separate from the QRS. The rate of a complete heart block is typically less than 60. Sinus bradycardia has a normal rhythm and a normal PR and QRS interval, but the rate is less than 60. Additional Info The PR interval represents the amount of time for atrial depolarization. The normal PR interval is 0.12 - 0.20 seconds. The normal QRS interval represents the time for ventricular depolarization. The normal is 0.04 - 0.12 seconds. When assessing a cardiac rhythm strip, the nurse should first determine the rate. The rate can be determined by examining a six-second strip, taking each QRS complex, and multiplying it by ten. The second step would be to determine the heart rhythm. To assess atrial regularity, use a pair of calipers to determine the distance between the P-P waves. Ventricular regularity may be assessed by using the calipers to measure the RR interval. The third step would be to analyze the P waves. The P waves should be present, occurring in regularity and in front of each QRS complex. Finally, the PR interval and QRS duration should be measured. The ST segment should then be analyzed, followed by the T-wave. The last measurement in this step would be assessing the QT interval. Last Updated - 03, Dec 2022

While monitoring a client with myocardial infarction, who is receiving tissue plasminogen activator (tPA), the nurse should plan to prioritize which of the following? A. Observe for neurological changes B. Monitor for any signs of renal failure C. Observe for signs of bleeding D. Check the client's food diary Submit Answer

Explanation Choice C is correct. Bleeding is the priority concern for any patient who is taking a thrombolytic medication. Choices A and B are incorrect. Although neurological status and renal function are monitored, they are not the primary concern. Choice D is incorrect. The client's food diary is not related to the use of this medication. NCSBN Client Need Topic: Physiological Integrity, Subtopic: Physiological Adaptation - Myocardial Infarction Last Updated - 02, Feb 2022

A client arrives at the emergency department (ED) via emergency medical services (EMS) after being hit by a car. The name of the client is unknown. The client is unconscious, having sustained a severe head injury and multiple fractures. An emergency craniotomy is indicated. Regarding informed consent for the surgical procedure, which of the following is the best action? A. Obtain a court order for the surgical procedure in place of an informed consent B. Ask the head of the EMS team to sign the informed consent C. Transport the client to the operating room for surgery immediately D. Call the police to report the incident, identify the client, and locate the family Submit Answer

Explanation Choice C is correct. Generally, a lack of decision-making capacity with inadequate time to find an appropriate proxy without harming the client, such as a life-threatening emergency where the client is not conscious, is a situation that allows informed consent to be waived. Here, the delay in providing an emergency craniotomy to an unconscious client would likely result in the client's death, making this case a situation where informed consent would be waived. Choice A is incorrect. Obtaining a court order for the necessary surgical procedure in place of informed consent is an unreasonable and lengthy intervention that would delay the required emergency treatment, likely leading to a negative patient outcome. Choice B is incorrect. Asking the head of the EMS team to sign an informed consent authorization is inappropriate, as this individual has no legal authority to authorize the unknown client to undergo an emergency craniotomy. Choice D is incorrect. Although a decision may be made to contact the police to report the incident, identify the client, and locate the family, at this point, the urgency will not allow for the surgery to be delayed to provide time for the police to find the client's family members. The client's severe head injury requires an emergent craniotomy. The longer the client is required to wait before surgery, the more likely the client is to have significant, long-term brain damage. Learning Objective Recognize that informed consent may be waived when a lack of decision-making capacity and inadequate time to find an appropriate proxy without harming the client exists. Additional Info In emergencies, when a decision must be made urgently, the client cannot participate in decision-making, and the client's surrogate is unavailable, the health care provider (HCP) may initiate treatment without prior informed consent. During these situations, the HCP should inform the client/surrogate at the earliest opportunity and obtain consent for ongoing treatment. Last Updated - 16, Jan 2022

After presenting with acute myocardial ischemia, a client was given 324 mg PO aspirin, three doses of 0.4 mg SL nitroglycerin tablets (taken five minutes apart), and oxygen via nasal cannula at 2L/minute. Which ECG change would indicate these interventions have been effective? A. Widening of the QRS complex B. Decrease in ectopic heartbeats C. ST-segment has returned to the baseline D. Reduction of the significant Q-wave Submit Answer

Explanation Choice C is correct. In myocardial ischemia, the ST-segment may appear elevated or depressed. In the presence of acute myocardial ischemia, ST-segment changes result from lack of oxygen to a specific region of the cardiac muscle. If treatment has been successful, the ST-segment will return to baseline. Choice A is incorrect. A widening of the QRS complex is not directly associated with myocardial ischemia. Choice B is incorrect. Ectopic heartbeats are changes in an otherwise normal heart rate leading to extra or skipped heartbeats. Although common, the cause of ectopic heartbeats is often idiopathic. The two most common types of ectopic heartbeats are premature ventricular contractions (PVCs) and premature atrial contractions (PACs). Overall, ectopic heartbeats are not directly associated with myocardial ischemia. Choice D is incorrect. The presence of a Q-wave is not directly associated with myocardial ischemia. Learning Objective Recognize that in a client with acute myocardial ischemia treated with nitroglycerin, aspirin, and oxygen, an ECG demonstrating the ST-segment has returned to baseline is indicative of the effectiveness of the interventions. Additional Info Nitroglycerine decreases cardiac oxygen demand. Unstable angina, NSTEMI, and STEMI represent worsening degrees of myocardial ischemia and necrosis; the distinctions help differentiate prognosis and guide treatment. Angina is the most common symptom of myocardial ischemia. Ischemia (i.e., lack of oxygen) that occurs with angina is limited in duration and does not cause permanent myocardial tissue damage. Under certain conditions, including myocardial ischemia (decreased blood flow), any cardiac cell may produce electrical impulses independently and create dysrhythmias. Once an artery becomes 50% occluded, blood flow is impaired, creating myocardial ischemia when myocardial demand is increased. Clients with unstable angina may present with ST changes on a 12-lead ECG but not have changes in troponin levels. Ischemia is present but is not severe enough to cause detectable myocardial damage or cell death. As troponins assays become more sensitive, the diagnosis of unstable angina is decreasing. Silent myocardial ischemia increases the incidence of new coronary events and should be treated aggressively. Last Updated - 03, Nov 2022

The nurse in the intensive care unit is caring for a patient that has left-sided heart failure with pulmonary edema as a complication. The nurse identifies a nursing diagnosis of impaired gas exchange related to fluid in the alveoli. Which of the following interventions would be considered the least priority according to the nursing diagnosis? A. Giving oxygen and watching for dry nasal mucus membranes. B. Placing the client in Fowler's position. C. Providing a pressure reducing mattress. D. Encouraging the client to turn, deep breathe, cough, and use the incentive spirometer. Submit Answer

Explanation Choice C is correct. Pressure reduction mattresses and beds are available to decrease the pressure on the sacrum when the client is in bed. Implementing measures to relieve sacral stress, however, is the least priority when managing clients in acute pulmonary edema. Choice A is incorrect. Oxygen therapy improves oxygenation by increasing the amount of oxygen available for delivery and can help relieve the client's dyspnea. Continuous oxygen administration can dry the patient's mucus membranes. This should be a priority intervention. Choice B is incorrect. This position facilitates the expansion of the diaphragm and should be a priority intervention. Choice D is incorrect. Turning, deep breathing, coughing, and the use of an incentive spirometer will all help clear the airways and facilitate oxygen delivery. Additional Info Last Updated - 31, Oct 2022

What should the nurse do during assessment when a patient reports swelling in his ankles? A. Measure his ankles at their widest point. B. Ask the patient to elevate his feet to better visualize his ankles. C. Press fingers in the edematous area to evaluate for a remaining indentation after the nurse removes his/her fingers. D. Evaluate further for brown hyperpigmentation that is associated with venous insufficiency. Submit Answer

Explanation Choice C is correct. Reports of swelling require evaluation for pitting edema. This can be done by pressing fingers in the edematous area to evaluate for a remaining indentation after removing one's fingers. Choice A is incorrect. The nurse would not measure his ankles at their widest point. Choice B is incorrect. The patient's ankles should be evaluated for pitting edema. Elevating his feet is an intervention to prevent the pooling of fluid. However, it is not part of the assessment for edema. Choice D is incorrect. Hyperpigmentation is an indication of late-stage chronic venous insufficiency. Assessing for hyperpigmentation is not an immediate assessment necessary for the report of swelling of the ankles. NCSBN Client Need Topic: Physiological Integrity, Subtopic: Physiological Adaptation; Assessing the Neurologic, Musculoskeletal, and Peripheral Vascular Systems Last Updated - 10, Feb 2022

The nurse assesses the following telemetry strip for a client on a medical-surgical unit. Based on the rhythm, what is the priority action for the nurse to take? See the exhibit. View Exhibit A. Prepare for synchronized cardioversion B. Obtain a prescription for intravenous (IV) atropine C. Review the most recent labs D. Ask the patient about palpitations Submit Answer

Explanation Choice C is correct. The EKG strip shows normal sinus rhythm with occasional premature ventricular contractions (PVCs). A PVC produces wide, bizarre complexes, with the P wave hidden within the QRS complex (not visible). After assessing the client, the nurse should review the client's most recent laboratory data because low levels of magnesium and potassium may cause PVCs. Choices A, B, and D are incorrect. Cardioversion would not be appropriate for this rhythm. Synchronized cardioversion delivers a timed electrical current to reset the heart's electrical activity and is indicated in tachydysrhythmias such as atrial fibrillation or atrial flutter. Atropine is indicated for increasing the heart rate, such as in the case of symptomatic bradycardia. This client's rhythm is 80 BPM, which is within the normal rate and does not require adjustment. This client may notice or feel palpitations due to the intermittent irregular beats, but this assessment data would not be an important aspect of determining the need for treatment and would not be a high priority. Additional Info PVCs are the most common type of arrhythmia, can occur in healthy individuals, and are typically not concerning in an otherwise normal rhythm. However, they can be a warning sign of electrolyte imbalance (hypokalemia, hypomagnesemia), hypoxemia, acid-base imbalances, or myocardial ischemia. PVC may also be triggered by excessive stress, caffeine use, nicotine, and sleeplessness. The nurse should assess the client's presentation and review the most recent lab values to determine if there is any apparent physiological cause for the arrhythmia. If the client presents without symptoms and if no abnormal labs are noted, the nurse should continue to monitor for any symptom development or changes in the rhythm. Last Updated - 27, Dec 2022

A labor and delivery nurse is caring for a G4P3 client currently in active labor. More specifically, the client is undergoing a trial of labor after cesarean(TOLAC). All three of the client's prior deliveries were cesarean sections. During the TOLAC, the client screams out, reporting severe abdominal pain, describing the pain as "ripping" or "tearing" in nature. Upon assessment, the client complains of sudden hypotension, lightheadedness, and dizziness. Fetal bradycardia is suddenly noted by the nurse. The charge nurse immediately tells the unit secretary to contact the provider on call. Which of the following should the nurse perform first? A. Prepare the client for an immediate vaginal delivery B. Start a second large bore IV C. Increase the rate of the client's intravenous fluids D. Assess the client's contraction pattern Submit Answer

Explanation Choice C is correct. The client's presentation is consistent with a ruptured uterus (sudden onset of severe pain and cessation of previously present uterine contractions). A uterine rupture is an emergency and must prompt immediate action. A delay in delivery, resuscitation, or surgery increases maternal and fetal risk. A uterine rupture will typically be associated with fetal bradycardia. Thus, the initial treatment is an emergent cesarean delivery. Immediate delivery and treatment of maternal hemorrhage are the two most critical interventions. Uterine structures are hypervascular during pregnancy, and a rupture puts the client at significant risk of severe hemorrhage, with hypotension and hemorrhagic shock to follow. The nurse should anticipate these complications. The nurse should increase the rate of IV fluids to counteract the hypovolemia due to blood loss. A blood transfusion should be considered for clients with significant blood loss. Choice A is incorrect. Due to the client's ruptured uterus, the client cannot deliver this baby vaginally or spontaneously due to the ruptured uterus. Treatment of uterine rupture is immediate laparotomy with cesarean delivery and, if necessary, hysterectomy. Choice B is incorrect. Although not the immediate priority, a uterine rupture requires simultaneous delivery and treatment of maternal hemorrhage. Placing a second large-bore intravenous line should is still a priority. If large-bore intravenous access cannot be obtained, central venous access with a large bore sheath introducer should be considered. Initial resuscitation is often provided by infusing Lactated Ringers electrolyte solution. Brisk and large volume blood loss should prompt early blood transfusion, so the nurse should ensure the client has been typed and crossed, applicable consents have been signed, and the blood bank has been made aware of the impending situation. Choice D is incorrect. Due to the rupture of the client's uterus, the contraction pattern will likely no longer be able to be assessed. Additionally, the total cessation of contractions on the client's exam further indicates a complete uterine rupture. Learning Objective Recognize the signs and symptoms of uterine rupture, understanding that a client with suspected uterine rupture requires immediate treatment, including delivery and treatment of maternal hemorrhage performed simultaneously. Additional Info Uterine rupture is the spontaneous tearing of the uterus that may result in the fetus being expelled into the peritoneal cavity. Although rare, uterine rupture typically occurs during late pregnancy or active labor. Symptoms and signs of uterine rupture include fetal bradycardia, variable decelerations, evidence of hypovolemia, loss of fetal station (detected during cervical examination), and severe or constant abdominal pain. The biggest risk factor for a uterine rupture is TOLAC. Most uterine ruptures occur during labor. Treatment of uterine rupture is immediate laparotomy with cesarean delivery and, if necessary, hysterectomy. The rate of uterine rupture is highly dependent upon the number of cesarean deliveries a woman has had and the type of uterine incision present. Last Updated - 23, Aug 2022

What medical device is displayed in the image below? Source : Archer Review A. A subcutaneous implantable cardioverter defibrillator B. A central venous catheter C. A cardiac pacemaker D. A chest tube drainage system Submit Answer

Explanation Choice C is correct. The image displayed depicts a cardiac pacemaker. Here, a biventricular pacemaker is shown. These types of pacemakers are used for cardiac resynchronization therapy and typically have three leads: one guided to the right ventricle, one to the left ventricle, and one to the right atrium. Choice A is incorrect. The image shown does not depict a subcutaneous implantable cardioverter defibrillator (S-ICD), as these devices are not implanted into the client's left upper chest. S-ICDs are placed through an incision site under the client's left arm, where they subsequently remain. Additionally, S-ICDs differ from pacemakers as S-ICD placement does not require lead placement within the heart. The device itself is placed subcutaneously under the skin along the rib cage inferior to the left axillary region, rather than in the standard location near the clavicle. The lead that is connected to the device is tunneled subcutaneously along the left side of the sternum rather than within the heart. The image below depicts the placement of a S-ICD. Source : Archer Review Choice B is incorrect. The image provided does not display a central venous catheter. Choice D is incorrect. A chest tube drainage system is not displayed in the image provided. Learning Objective Identify the displayed medical device as a cardiac pacemaker. Additional Info Treatment of arrhythmias depends on the symptoms and the severity of the arrhythmia. Pacemakers sense electrical events and respond when necessary by delivering electrical stimuli to the heart. There are three basic kinds of pacemakers: Single chamber [one lead attached to the upper or lower cardiac chambers] Dual-chamber [two leads, with one for both the upper and lower chambers] Biventricular pacemakers Permanent pacemaker leads are placed via thoracotomy or transvenously, but some temporary emergency pacemaker leads can be placed on the chest wall. There are numerous indications for pacemaker placement, but most typically involve symptomatic bradycardia or high-grade atrioventricular block. Last Updated - 04, Oct 2022

You are working in an adult telemetry step-down unit and have five patients to manage. You see the following rhythms on the monitor from your patients. Which patient should you assess first? Select the image of the ECG for the patient you would assess first. A . B . C . D . Submit Answer

Explanation Choice C is correct. This patient is showing sustained ventricular tachycardia on the monitor. This is a fatal rhythm and the nurse must immediately assess the patient as they could quickly arrest and necessitate a code blue. This is the correct patient to assess first. Choice A is incorrect. This patient is in normal sinus rhythm. They are not the priority for assessment. Choice B is incorrect. This patient is demonstrating normal sinus rhythm with one unifocal premature ventricular contraction (PVC). While the nurse does need to assess them, they are not the priority. PVCs can be well-tolerated and a singular one is not immediately dangerous. Choice D is incorrect. This patient is in sinus tachycardia. This could be caused by a fever, dehydration, or could be the patient's baseline. While the nurse should assess the patient and determine the cause of the tachycardia, he is not the priority of the five patients shown. NCSBN Client Need Topic: Reduction of Risk Potential Subtopic: Diagnostic Tests Last Updated - 26, Dec 2021

A client admitted due to emergent hypertension is about to be discharged. The nurse is giving instructions about dietary modifications. Which food choice by the client would indicate an accurate understanding of a low fat, low cholesterol diet? A. Macaroni and cheese B. Fish and chips C. Turkey breast salad D. Pepperoni pizza Submit Answer

Explanation Choice C is correct. Turkey breast contains 2.2 grams of saturated fats, 2.1 grams of polyunsaturated fats, and 2.6 grams of monounsaturated fats per 100 grams. It is also a high protein source `with 29 grams per 100 grams. Choice A is incorrect. Macaroni and cheese contain 8 grams of cholesterol and 5 grams of fat per 100-gram servings. Choice B is incorrect. Fish and chips contain an average of 11.1 grams of fat per 100-gram serving. Choice D is incorrect. Pepperoni pizza contains 10 grams of fat per 100-gram serving. It is also high in saturated fat. Additional Info Source : Archer Review Last Updated - 11, Aug 2021

What is the correct documentation of the patient's peripheral pulse when the finding is that the posterior tibial pulse is weak and thready? Correct A. Grade C posterior tibial pulse [9%] B. Posterior tibial pulse is Grade B [13%] C. The client's posterior tibial is 2 [19%] D. Posterior tibial pulse is 1

Explanation Choice D is correct. When assessing pulses, the strength, volume, and fullness of the peripheral pulses are categorized and documented as follows: 0: Absent pulses 1: Weak pulse 2: Normal pulse 3: Increased volume 4: Abounding pulse Choices A and B are incorrect. Grades and grading are not used to document pulses. Choice C is incorrect. The pulse is weak and thready, not regular. NCSBN Client Need Topic: Physiological Integrity, Subtopic: Vital Signs Last Updated - 16, Feb 2022

The nurse is discussing cardiac dysrhythmias with a student. Which of the following statements, if made by the student, would indicate effective understanding? A. "Atrial fibrillation may cause venous thromboembolism." B. "Immediate defibrillation is needed for asystole." C. "Cardioversion may be used for ventricular fibrillation." D. "Adenosine is used to treat supraventricular tachycardia." Submit Answer

Explanation Choice D is correct. Adenosine is the initial medication utilized to manage supraventricular tachycardia (SVT). This medication slows the conduction time through the AV node and restores normal sinus rhythm. Choices A, B, and C are incorrect. Atrial fibrillation does lead to a potential thrombus that may evolve into an embolism; however, it does not cause venous thromboembolism because the blood pooling in the left atrial appendage gets ejected into the arterial circulation, not venous. Thus, the most likely complication is an embolic stroke. Asystole is treated with high-quality CPR and IV epinephrine. Defibrillation is not indicated for asystole. Ventricular fibrillation responds to defibrillation, not cardioversion. Defibrillation delivers a higher amplitude of joules and does not need to be synchronized. Cardioversion can only be done if the client has a pulse. Additional Info Supraventricular tachycardia (SVT) is a tachycardia that is caused by an impulse recirculating throughout the atrial pathway. Adenosine is used to treat SVT; before giving this medication, the nurse must have the client connected to a cardiac monitor and have a defibrillator as well as a crash/code cart nearby. The client should have the pads affixed to them just in case the medication causes a prolonged period of asystole. The medication must be pushed by rapid IV push along with an immediate flush of 0.9% saline. Last Updated - 16, Feb 2022

When the nurse notes an irregular radial pulse in a client, further evaluation should include assessing for which of the following? A. The carotid pulse B. Diminished peripheral circulation C. The brachial pulse D. A pulse deficit Submit Answer

Explanation Choice D is correct. Assessing for a pulse deficit provides an indirect evaluation of the heart's ability to eject enough blood to produce a peripheral pulse. When a pulse deficit is present, the radial pulse is less than the apical pulse. Choices A and C are incorrect. If the pulse is irregular, the correct protocol is to assess for a pulse deficit, which means measuring the apical and radial pulses simultaneously. Choice B is incorrect. Diminished peripheral circulation is not the correct assessment. NCSBN Client Need Topic: Physiological Integrity, Subtopic: Reduction of Risk Potential; Pulse Last Updated - 03, Feb 2022

The nurse monitoring the cardiac monitor notes the absence of P waves and fibrillations before each QRS complex. How should the nurse classify this rhythm? A. Sinus tachycardia B. Ventricular fibrillation C. Ventricular tachycardia D. Atrial fibrillation Submit Answer

Explanation Choice D is correct. Atrial fibrillation is associated with the absence of P waves and fibrillations before the QRS complexes. Choice A is incorrect. There is no information in this question that indicates tachycardia because the heart rate is not included. Choice B is incorrect. The loss of QRS complexes categorizes ventricular fibrillation. Choice C is incorrect. There is no information in this question that indicates tachycardia because the heart rate is not included. NCSBN client need Topic: Oncology or Maternal Infant Health Additional Info Last Updated - 25, Jan 2022

The cardiac nurse is evaluating cardiac markers to determine whether or not their patient's heart has suffered from muscle damage. The nurse is aware if damage has occurred, CK-MB levels will be their highest after how many hours? A. 3 to 6 B. 1 to 2 C. 48 to 72 D. 18 Submit Answer

Explanation Choice D is correct. CK-MB, or creatine kinase myocardial muscle, levels measure muscle cell death and are at their highest elevation 18 hours after cardiac muscle damage. CK-MB levels first begin elevating about 3 to 6 hours after a cellular injury or myocardial infarction and stay elevated for about 48 to 72 hours. Choice A is incorrect. While CK-MB levels begin to rise about 3 to 6 hours after myocardial cellular death, they do not peak until 18 hours. Choice B is incorrect. CK-MB enzyme levels will not have risen yet by 1 to 2 hours. Standards do not begin to rise until 3 to 6 hours and hit their peak around 18 hours. Choice C is incorrect. At 48 to 72 hours, CK-MB enzyme levels will likely return to normal. NCSBN client need Topic: Physiological adaptation, reduction of risk potential Last Updated - 11, Dec 2021

A 45-year-old man is admitted to the telemetry unit for observation. After waking up in the morning, the client asks the nurse if there is any coffee. What should be the nurse's best response to the client? A. "Hot beverages are not allowed because of the condition of your heart." B. "Coffee was not in your diet order by the physician" C. "We don't have coffee in the unit. I can bring you some tea if you like." D. "As of the moment you cannot have coffee because it has caffeine, and caffeine can affect your heart." Submit Answer

Explanation Choice D is correct. Coffee contains caffeine that is a stimulant. It causes vasoconstriction and increased blood pressure. Thus, it should be avoided by clients who have dysrhythmias. Choice A is incorrect. There is no evidence that hot beverages are not suitable for the heart. Choice B is incorrect. Even if caffeine is ordered by the doctor to be avoided, the nurse should provide a much more comprehensive explanation to the patient as to why he should not drink coffee. Choice C is incorrect. Tea also contains caffeine and should also be avoided by the client. Last Updated - 15, Feb 2022

When compared to younger adults, the nurse recognizes that the older clients have variations in pulse with: A. Food intake B. Heat C. Respirations D. Exercise Submit Answer

Explanation Choice D is correct. Exercise increases the heart rate because of increased metabolic demands. Aging adults have a normal pulse range of 60-100 beats/minute. However, the maximum heart rate in older adults is much lesser with exercise. In older adults, the radial artery may stiffen from peripheral vascular disease. With exercise, a variation in the pulse is noted in older adults compared to younger adults. The pulse rate of older adults takes longer to rise to meet sudden increases in demand, takes longer to return to resting state, and tends to be lower than that of younger adults. Choices A and B are incorrect. Certain types of food may cause changes within the body (such as salty foods can increase blood pressure and affect heart rate). Also, internal temperature changes may cause an increase in heart rate. However, overall food intake and heat are not causes for variations in pulse. Choice C is incorrect. Sinus arrhythmia, a variation in pulse with respiration, is common among children, not older adults. NCSBN Client Need Topic: Health Promotion and Maintenance, Subtopic: Pulse Last Updated - 14, Feb 2022

A client is being discharged following the insertion of a permanent pacemaker. Which of the following should be included in the client's discharge instructions? A. Air travel will not be possible due to airport screening equipment. B. You will need to discard any radios at home that have antennas. C. Computed tomography (CT) scans are not permitted with this device. D. You should use your cellular phone on the opposite side of the generator. Submit Answer

Explanation Choice D is correct. For a client with a pacemaker, it is recommended that they talk on their cellular phone opposite of the pulse generator to prevent electromagnetic interference. Choices A, B, and C are incorrect. Air travel is not prohibited for a client with a pacemaker. They should carry their medical alert card if they are stopped by airport security. Discarding radios and other home appliances is unnecessary as they do not cause any interference. CT scans are permitted for a client with a pacemaker. Diagnostics with an MRI are contraindicated. Additional Info Following the placement of a pacemaker, the client will be provided with the following instructions: · Keep a pacemaker identification card in your wallet. · Following the procedure, avoid lifting your arm over your head for four weeks. · Do not lift heavy objects following the procedure until cleared by your primary healthcare provider (PHCP). · Taking a bath and shower is not restricted. · Do not apply pressure over the generator, and do not wear any tight clothing. · Operating household appliances would be considered safe.

The patient is diagnosed with atrial fibrillation. What assessment data would require immediate intervention by the RN? A. Irregular QRS complexes on telemetry reading B. Rapid, irregular pulse C. The patient reports palpitations D. The patient reports lightheadedness Submit Answer

Explanation Choice D is correct. Lightheadedness/dizziness may be a sign that the patient's rhythm has changed. The nurse should assess the patient and the rhythm as well as report any changes to the physician. Choice A is incorrect. Irregular QRS complexes are expected in atrial fibrillation. Choice B is incorrect. Irregular and rapid pulse readings are expected in atrial fibrillation. Choice C is incorrect. A patient may be able to feel palpitations when in atrial fibrillation. NCSBN Client Need Topic: Cardiovascular, Subtopic: Establishing priorities, diagnostic tests, the potential for alterations in body systems, changes/abnormalities in vital signs Additional Info Source : Archer Review Last Updated - 14, Feb 2022

The nurse is caring for a client with heart failure. The patient is ordered a nitroglycerin patch to be attached. Which of the following nursing actions regarding the administration of a nitroglycerin patch is most relevant? A. Use a bare hand when putting the patch on the patient. B. Place the patch on the same spot every day. C. Place the client in the supine position with his feet elevated on a pillow. D. Instruct the client to rise slowly. Submit Answer

Explanation Choice D is correct. Patients under nitroglycerin therapy are at risk for postural hypotension. The client should rise slowly to avoid a sudden drop in blood pressure when standing up. Choice A is incorrect. The nurse should wear gloves when administering a nitroglycerin patch to avoid skin contact with the medication. Choice B is incorrect. The patch should be rotated to ensure optimum absorption through the skin. Choice C is incorrect. The purpose of nitroglycerin in heart failure is to dilate the venous circulation and trap the blood in the veins, decreasing the preload. Placing the client in the supine position and elevating his feet increases venous return, thus increasing preload. This defeats the purpose of nitroglycerin. Last Updated - 16, Oct 2021

The patient has just been diagnosed with hyperlipidemia. Aside from the prescribed atorvastatin, she is advised to lose weight and implement some dietary changes. The nurse talks to her about possible nutritional modifications. All of the following are accurate, except: A. Replace vegetable oil with canola oil when cooking. B. Eat fish like tuna and salmon more often. C. Eat more fruits and vegetables. D. Eat red meats more often. Submit Answer

Explanation Choice D is correct. Red meats are rich in saturated fat. It should be eaten less frequently because it contributes to high cholesterol levels. Choice A is incorrect. Replacing vegetable oils high in polyunsaturated fats with canola oil (monounsaturated fats) is more beneficial in reducing cholesterol levels. Choice B is incorrect. Fish like tuna and salmon are rich in omega 3 fatty acids, which help in reducing harmful cholesterol levels. Choice C is incorrect. Fruits and vegetables contain fiber, which promotes a healthy cholesterol level. Last Updated - 22, Jan 2022

The nurse is talking to the patient's son who was just diagnosed with coronary heart disease. He asks about the risk factors that can be modified to decrease the chances of acquiring this medical condition. The nurse educates him by saying that the following are modifiable risk factors for coronary heart disease: A. Gender, cholesterol levels, and obesity B. Age and elevated blood pressure C. Stress, age, and gender D. Smoking, obesity, and physical activity Submit Answer

Explanation Choice D is correct. Smoking, obesity, and physical activity are all modifiable risk factors of coronary heart disease. Choice A is incorrect. Gender is a non-modifiable risk factor, whereas cholesterol levels and obesity are modifiable risk factors. Choice B is incorrect. Age is a non-modifiable risk factor, whereas blood pressure is a modifiable risk factor. Choice C is incorrect. Stress is a contributing risk factor, whereas age and gender are non-modifiable risk factors. Last Updated - 16, Oct 2021

The emergency department nurse is caring for a client with congestive heart failure who reports dyspnea and a persistent cough. The nurse obtains the client's vital signs and suspects that the client is experiencing which condition? See the image below. A. Pulmonary embolism B. Hypovolemic shock C. Disseminated intravascular coagulation (DIC) D. Pulmonary edema Submit Answer

Explanation Choice D is correct. The client's history of congestive heart failure significantly increases the risk for pulmonary edema. The vital signs show respiratory distress (tachypnea, hypoxia, and tachycardia), which supports the complication of pulmonary edema. Choices A, B, and C are incorrect. These conditions are not as likely to occur in an individual with CHF. While CHF places a client at higher risk for venous thromboembolism, it is reasonable for the nurse to first suspect the most common complication, which is pulmonary edema. DIC is quite rare and is highly unlikely. Hypovolemic shock is not plausible because the client with CHF typically has a problem with fluid volume excess, not a deficit. Additional Info Pulmonary edema is a medical emergency requiring prompt recognition and aggressive management through oxygenation, nitrates, and intravenous diuretics. Manifestations of pulmonary edema include crackles, dyspnea at rest, orthopnea, frequent coughing, hypoxia, tachypnea, anxiety, and a feeling of impending doom. Last Updated - 05, Jul 2022

A middle-aged African American is being treated in the emergency department (ED) for an acute sickle cell crisis. Which of the following should the nurse use when positioning the client to facilitate oxygenation and adequate circulation? A. Side-lying with flexed knees B. Fetal position C. Semi-Fowler's position with knees and hips bent D. Semi-Fowler's with legs extended on the bed Submit Answer

Explanation Choice D is correct. The nurse should place the client in Semi-Fowler's position with his extremities relaxed and straightened, as clients in a sickle cell crisis should be positioned to optimize circulation and oxygenation. Choice A is incorrect. The nurse's goal should be to facilitate oxygenation and promote adequate circulation for the client. Knee flexion would impede the flow of both circulation and oxygenation of the client. Choice B is incorrect. In a fetal position, the client's knees and hips are flexed. Knee and hip flexion would impede circulation in this patient. The nurse should ensure that flow is facilitated, not hindered, when positioning the client. Choice C is incorrect. Semi-Fowler's position facilitates lung expansion; however, the bent knees and hips restrict the client's circulation. The nurse should not place the client in this position. Learning Objective Identify the correct position for a client experiencing an acute sickle cell crisis as a semi-Fowler's position with legs extended on the bed. This position facilitates oxygenation and adequate circulation. Additional Info Sickle cell disease is an inherited blood disorder marked by flawed hemoglobin. In the United States, sickle cell disease mainly affects African Americans. Clients with sickle cell disease have abnormal hemoglobin, which is "crescent" or "sickle"-shaped in nature, thus inhibiting these cells' oxygen-carrying capacity. Last Updated - 26, Jan 2022

During a client's bronchoscopy, the nurse notices the client necessitates the use of a rigid scope. During the insertion of the scope by the health care provider (HCP), the nurse notes the client is experiencing a vasovagal response when noticing which of the following? A. Dilated pupils B. Bronchodilation C. Decrease in gastric secretions D. Noticeable drop in heart rate Submit Answer

Explanation Choice D is correct. The nurse would notate a vasovagal response upon seeing a sudden, noticeable drop in the client's heart rate. Here, during the bronchoscopy, the involvement of a foreign object (i.e., the HCP's scope) in the client's pharynx likely caused vagus nerve stimulation. This stimulation resulted in a vasovagal response by the client, manifested by a sudden decrease in the client's heart rate. Choice A is incorrect. Stimulation of the vagus nerve does not cause dilation of the pupils. Choice B is incorrect. Stimulation of the vagus nerve does not cause bronchodilation. Choice C is incorrect. During vagus nerve stimulation, an increase, not decrease, in gastric secretions is typically noted. This response occurs as a result of increased parasympathetic activity. Learning Objective Identify a sudden, noticeable drop in heart rate as a symptom found in a client actively experiencing a vasovagal response. Additional Info Increased vagal firing (i.e., increased parasympathetic activity) at the sinus node and the atrioventricular node causes a decrease in heart rate. This decrease in heart rate can be profound, with asystole lasting as long as several seconds. Vagal stimulation may result in bradycardia, hypotension, heart block, ventricular tachycardia, or other dysrhythmias. Last Updated - 03, Nov 2022

The nurse is talking to an elderly client who is being discharged with digoxin as a take-home medication. The nurse should initiate further teaching when the client states: A. "I won't take the tablet when my pulse is too slow." B. "I guess I'll be eating a lot of spinach and bananas from now on." C. "So, I need to watch out for any nausea and vomiting every now and then." D. "It's good that I don't have to get my blood examined for potassium from now on." Submit Answer

Explanation Choice D is correct. This is an inaccurate statement by the client and necessitates the nurse to provide additional clarification. Hypokalemia ( low potassium level) increases the risk of digoxin toxicity, especially cardiac arrhythmias. The client needs to understand that serum potassium levels need to be monitored frequently while on Digoxin. Choice A is incorrect. Digoxin can cause bradycardia. Significantly reduced heart rate below the baseline may indicate supra therapeutic digoxin level. Checking his pulse and not taking the Digoxin when his pulse falls below 50-60 beats per minute is correct about the medication. Choice B is incorrect. Spinach and bananas are foods rich in potassium. Elderly patients are at risk of hypokalemia due to inadequate oral intake. Many patients on Digoxin are also on multiple other medications such as diuretics which increase the risk of hypokalemia. Low potassium level increases the risk of Digoxin toxicity. Consuming high potassium foods would be an effective way to prevent hypokalemia. This statement indicates an accurate understanding. Choice C is incorrect. The patient needs to recognize signs related to digoxin toxicity to access medical help immediately. Gastrointestinal side effects such as nausea and vomiting may be an early sign of digoxin toxicity. Learning Objective Recognize that digoxin toxicity increases in the presence of hypokalemia, and healthcare providers should monitor labs, especially when the clients are on other concomitant medications such as loop diuretics. Last Updated - 11, Dec 2021

The nurse is caring for a client with the below tracing on the electrocardiogram (ECG). It would be correct for the nurse to document this tracing as See the image below. A. sinus bradycardia. B. normal sinus rhythm with first degree block. C. atrial flutter. D. atrial fibrillation. Submit Answer

Explanation Choice D is correct. This tracing reflects atrial fibrillation. Atrial fibrillation characteristically has no definitive P-waves because fibrillatory waves replace them before each QRS. Choices A, B, and C are incorrect. This tracing does not reflect these options because of the absence of P-waves, irregularity, and fibrillatory waves before the QRS making it atrial fibrillation. Additional Info Atrial fibrillation is associated with atrial fibrosis and loss of muscle mass. These structural changes are common in heart diseases such as hypertension, heart failure, and coronary artery disease. Characteristically, atrial fibrillation is irregularly, irregular with no P-waves identified. The biggest complication associated with atrial fibrillation is stroke because of blood pooling in the atrium. Treatment options for atrial fibrillation include digoxin (not as commonly used), amiodarone, diltiazem, verapamil, or atenolol. The client may be prescribed an oral anticoagulant such as apixaban to prevent thrombosis. If medication is not desired, synchronized cardioversion may be prescribed. Last Updated - 15, Sep 2022

The nurse is caring for a client with the below tracing on the electrocardiogram (ECG). The client reports dyspnea and dizziness. The nurse should obtain a prescription for which medication? See the image below. A. Diltiazem B. Amiodarone C. Labetalol D. Atropine Submit Answer

Explanation Choice D is correct. This tracing reflects sinus bradycardia. Atropine would be clinically indicated if the client was experiencing symptomatic bradycardia. If the client has bradycardia and concurrent dizziness, hypotension, or dyspnea, this calls for treatment with atropine. Atropine must be administered while an emergency (code) cart is nearby. Choices A, B, and C are incorrect. These options would be highly detrimental if a client had bradycardia because these medications decrease the heart rate, and doing so would be catastrophic. Diltiazem is commonly utilized in the management of atrial fibrillation Amiodarone is an umbrella drug used in the treatment of ventricular tachycardia, ventricular fibrillation, and atrial fibrillation Labetalol is one of the drugs of choice for a client experiencing a hypertensive crisis. This medication lowers both heart rate and blood pressure. Additional Info Sinus bradycardia is a regular rhythm with a rate of less than sixty. Sinus bradycardia is only concerning if the client is symptomatic. Pathological causes of bradycardia include severe hypothyroidism, hypothermia, anorexia nervosa, and prolonged hypoxia. For a client receiving intravenous atropine, the nurse should monitor the heart rate and rhythm after administration. An increased heart rate is expected. Last Updated - 27, Jun 2022

The nurse reviews the client's continuous telemetry monitor and observes the following. As the nurse reviews the client's current medications, which prescribed medication is most likely causing this tracing? See the image below. A. Losartan B. Nitroglycerin transdermal patch C. Clonidine D. Verapamil Submit Answer

Explanation Choice D is correct. This tracing reflects sinus bradycardia. Verapamil is a calcium channel blocker, and a property unique to verapamil is that it decreases both blood pressure and heart rate. Verapamil may be indicated for the prevention of migraine headaches, hypertension, or vascular spasms. Choices A, B, and C are incorrect. Losartan is an ARB and may be used for hypertension or congestive heart failure. This medication does not lower heart rate. Nitroglycerin via a transdermal patch would increase the heart rate because of the reflex tachycardia it causes as it decreases blood pressure. Clonidine is an effective agent in treating hypertension and does not lower heart rate. Additional Info Sinus bradycardia is a regular rhythm with a rate of less than sixty. Sinus bradycardia is only concerning if the client is symptomatic. Pathological causes of bradycardia include severe hypothyroidism, hypothermia, anorexia nervosa, and prolonged hypoxia. Last Updated - 28, Jun 2022

The nurse is assessing a client with peripheral arterial disease (PAD). Which of the following findings would the nurse expect to observe? Select all that apply. A. Decreased peripheral pulses B. Pain with ambulation C. Reddish-brown ankle discoloration D. Bilateral dependent edema E. Protruding veins in the leg Submit Answer

Explanation Choices A and B are correct. Peripheral arterial disease (PAD) is characterized by atherosclerosis in the lumen of the peripheral arteries. PAD symptoms include pain in the extremities that may be exacerbated by walking and are relieved by rest (claudication). Decreased peripheral pulses are a consistent manifestation of PAD. Choices C, D, and E are incorrect. Hyperpigmentation of the ankles with edema is a finding consistent with venous insufficiency. Other features of venous insufficiency include protruding veins in the leg and telangiectasias. Peripheral pulses are typically regular in venous insufficiency. Edema is not associated with PAD because the issue is with an impediment in distal blood flow. When blood flow cannot return because of venous insufficiency, this causes the client to develop pedal edema. Additional Info Nursing care for those with PAD includes: Mitigate factors such as appropriate blood pressure control, avoiding atherogenic foods, and smoking cessation. Having the legs dependent helps facilitate blood flow. The nurse should educate the client to avoid applying heat to the affected extremity. The client should also avoid using constrictive clothing, garments, or devices. Medical management includes pharmacotherapy, including cilostazol, clopidogrel, and pentoxifylline. Peripheral artery disease: Last Updated - 07, Feb 2022

Which of the following would cause an increase in cardiac output? Select all that apply. A. 2 L normal saline fluid bolus B. Furosemide C. Propranolol D. Dopamine Submit Answer

Explanation Choices A and D are correct. Any increase in volume will cause an increase in cardiac output. When you increase the amount of mass in circulation, you increase the patient's stroke volume. Since the formula for cardiac output is CO = HR x SV, there are two ways to increase CO - by increasing the HR or increasing the SV. One sure way to increase the stroke volume, or amount of blood that the heart is pumping out with each beat, is to increase the amount circulating. Fluid boluses are commonly used to increase cardiac output (Choice A). Dopamine will increase cardiac output. Dopamine is an inotrope that improves the contractility of the heart. This means that the center will contract harder and pump out more blood with each contraction. This is an increase in stroke volume and because CO = HR x SV, an increase in SV causes an increase in CO. Any inotrope that improves the contractility of the heart will cause an increase in CO. This includes dopamine, dobutamine, and milrinone, to name a few (Choice D). Choice B is incorrect. Furosemide administration would decrease cardiac output. Furosemide is a potent loop diuretic, which induces diuresis and therefore reduces the amount of fluid in the vasculature. With reduced volume, preload in the heart is decreased. With decreased preload, there is diminished contractility due to Starling's law ("The greater the stretch on the myocardium before systole (preload), the stronger the ventricular contraction"). With decreased contraction, there is reduced stroke volume, and therefore decreased cardiac output. Choice C is incorrect. The administration of propranolol will decrease cardiac output. This is due to propranolol decreasing the heart rate. Propranolol is a beta-blocker used to control the pulse of the heart and therefore reduces the heart rate. Since CO = HR x SV, any decrease in the heart rate will decrease cardiac output; this is why the administration of any beta-blocker will lower cardiac output. NCSBN Client Need: Topic: Physiological Integrity, Subtopic: Physiological adaptation; Cardiac Additional Info Source : Archer Review Last Updated - 10, Nov 2021

The nurse is caring for a client with the following tracing on the electrocardiogram (ECG). The client has an oral temperature of 101 o F (38.3o C). The nurse should be prepared to obtain which prescription from the primary healthcare provider (PHCP)? Select all that apply. See the image below. A. 0.9% saline bolus B. Enalapril C. Levothyroxine D. Metoclopramide E. Acetaminophen Submit Answer

Explanation Choices A and E are correct. This reflects sinus tachycardia (ST). ST can be caused by various conditions, such as a febrile illness likely to induce dehydration. The appropriate action for the nurse is to obtain prescriptions for fluids to rehydrate the client and acetaminophen to mitigate the fever. Choices B, C, and D are incorrect. Enalapril is an ACE inhibitor that may decrease the client's blood pressure but would not directly impact the client's heart rate. Levothyroxine would worsen tachycardia because levothyroxine increases thyroid hormone, thereby increasing metabolic and sympathetic processes. Metoclopramide is a medication that causes gastric emptying and is not related to the treatment of ST. Additional Info ✓ Sinus tachycardia is regular rhythm, with a p-Wave present. ✓ The rate of sinus tachycardia is over 100 beats per minute. ✓ Causes of ST include anxiety, pain, fear, hyper- and hypoglycemia, and a febrile illness. ✓ Treatment of ST is the underlying cause; fluids need to be replaced if the client is hypovolemic. ✓ ST may be worrisome as it is one of the earliest manifestations of shock. Last Updated - 14, Jan 2023

The nurse is assessing a client with infective endocarditis (IE). Which of the following would be an expected finding? Select all that apply. A. Fever B. Night sweats C. Osler nodes D. Cardiac murmur E. Syncope F. Weight loss Submit Answer

Explanation Choices A, B, C, D, and F are correct. Infective endocarditis (IE) is a serious condition that, if untreated, may lead to heart failure. Infectious symptoms are the hallmark of this condition, including fever, night sweats, chills, weight loss, headache, and malaise. Other physical manifestations that may be assessed with IE include a cardiac murmur, Janeway lesions (flat, reddened maculae on hands and feet), Roth spots (hemorrhagic lesions that appear as round or oval spots on the retina), and Osler nodes (on palms of hands and soles of feet). Choice E is incorrect. Syncope is not a clinical feature of IE. This would be associated if the client was to have a vasovagal reaction. Additional Info Infective endocarditis occurs primarily in patients with injection drug use (IDU) and those who have had valve replacements, have experienced systemic alterations in immunity, or have structural cardiac defects. This is a condition caused by the invasion of bacteria which enter the client through contaminated needles, oral cavity following dental procedures, and/or skin abscesses. Classic manifestations of IE include • Fever associated with chills, night sweats, malaise, and fatigue • Anorexia and weight loss • Cardiac murmur (newly developed or change in existing) • Development of heart failure • Evidence of systemic embolization • Petechiae • Splinter hemorrhages • Osler nodes (on palms of hands and soles of feet) • Janeway lesions (flat, reddened maculae on hands and feet) • Roth spots (hemorrhagic lesions that appear as round or oval spots on the retina) Positive blood cultures Treatment of IE is antibiotic therapy for several weeks. Last Updated - 15, Jul 2022

Which of the following rhythm changes will have an impact on cardiac output? Select all that apply. A. Supraventricular tachycardia B. Sinus bradycardia C. Ventricular tachycardia D. Mobitz type II heart block Submit Answer

Explanation Choices A, B, C, and D are all correct. All rhythm changes will affect cardiac output. This is especially important to remember when you are administering antiarrhythmics to your patient, as these medications and their effect will also change the cardiac output. There are two reasons that rhythm changes affect cardiac output. 1 - they break your heart rate. 2 - they change your stroke volume. Remember, CO = HR x SV, so any change to either heart rate or stroke volume subsequently affects your cardiac output. A - Supraventricular tachycardia (SVT) - There is an increase in heart rate, but a decrease in stroke volume. This is because the heart is beating so fast that there is not enough time for diastole and, therefore, not enough time for the atriums to fill with blood. This decreases preload, which decreases contractility due to starling's law, which reduces stroke volume. Patients in SVT have decreased cardiac output. B - Sinus bradycardia - The heart rate is lower, due to bradycardia, so the cardiac output is lowered. Remember, CO = HR x SV. Decreased HR = decreased CO. C - Ventricular tachycardia - There is an increase in heart rate and a decrease in stroke volume. This is because the heart is beating fast and irregularly. There is not enough time for diastole and therefore not enough time for the atriums to fill with blood. This decreases preload, which decreases contractility due to Starling's law, which reduces stroke volume. Patients in VT have decreased cardiac output. This is a lethal rhythm. D - Mobitz type II heart block - This type of heart block causes a decreased heart rate, which once again decreases cardiac output. CO = HR x SV. Decreased HR = decreased CO. Patients in Mobitz type II heart block have decreased cardiac output. NCSBN Client Need: Topic: Physiological Integrity, Subtopic: Pharmacological therapies; Cardiac Additional Info Source : Archer Review

The nurse is caring for a client with systolic heart failure. Which of the following heart sounds would the nurse expect to auscultate? Select all that apply. A. S1 B. S2 C. S3 D. S4 Submit Answer

Explanation Choices A, B, and C are correct. S1 (Choice A) and S2 (Choice B) are normal heart sounds. These normal heart sounds would still be auscultated in a patient with heart failure. S1 is a benign heart sound caused by the closure of the mitral and tricuspid valves. S2 is a benign heart sound produced by the closure of the aortic and pulmonic valves. S3 ("ventricular gallop") and S4 ("atrial gallop") are abnormal heart sounds that can be auscultated in heart failure. Both of these heart sounds are low-pitched and best heard at the apex with the patient in the left lateral decubitus position. While S3 may sometimes be heard in healthy hearts (normal in children, pregnant women, and trained athletes), S4 is almost always abnormal. While S3 is a sign of systolic heart failure, S4 is heard in diastolic heart failure. It is essential to understand these two types of congestive heart failure (CHF) before discussing how S3 and S4 are produced. The nurse would expect to hear an S3 heart sound (Choice C) in her patient with systolic heart failure. S3 occurs after S2 with the opening of the mitral valve, and a passive flow makes the sound of a large amount of blood hitting a compliant left ventricle. This large amount of blood hitting the left ventricle is because of the underlying fluid volume overload seen with systolic heart failure. Choice D is incorrect. The nurse would not expect to hear an S4 heart sound in her patient with systolic heart failure. This is seen in diastolic heart failure. S4 occurs before S1 when the atria contract to actively force blood into the left ventricle. A stiff, non-compliant left ventricle causes it. When the atrial contraction forces blood through the mitral valve, the force is increased due to resistance offered by a stiff ventricle - in this scenario, S4 is caused by the blood striking the left ventricle. S4 heart sounds can also be heard in active ischemia. Please note that if a patient has atrial fibrillation and the atria are not contracting, it is impossible to have an S4 heart sound.

A 45-year old male presents to the emergency department with severe diaphoresis, nausea, shortness of breath, and left-sided neck and chest pain that started 30 minutes prior while doing yard work. What actions should the triage nurse perform immediately for this patient? Select all that apply. A. Start an IV and obtain a troponin B. Call the cardiologist C. Administer 325 mg aspirin D. Give 1 tablet of sublingual nitroglycerin E. Inquire about previous cardiac history F. Perform an EKG Submit Answer

Explanation Choices A, C, D, E, and F are correct. This patient is experiencing subtle signs that could be indicative of myocardial infarction (MI). Starting an IV and obtaining a troponin (Choice A) is especially crucial for the future administration of drugs and assessing the chemical damage done to the heart. Drugs that could be ordered by the physician if the patient is having an MI are heparin, ticagrelor, and aspirin. Administering 325 mg aspirin (Choice C) is a gold standard for angina management due to its blood-thinning effects and minimal side effects. Nitroglycerin (Choice D) is also essential to administer to a patient with angina, as long as the patient's blood pressure is above 100/60 mmHg. This medication allows the blood vessels to dilate (reduces preload), which will allow more blood flow to the heart. Asking about previous cardiac history (Choice E) is essential in obtaining further information about this patient. This will help evaluate the patient's cardiac risk and the risk of having an acute event now. Performing an EKG (Choice F) is essential because it will allow the emergency physician to diagnose the patient with a STEMI or NSTEMI and the right treatment. Choice B is incorrect. Calling the cardiologist would be premature at this point. The emergency department physician will evaluate the EKG for any changes and contact the cardiologist if necessary. Calling the cardiologist down to the emergency department is essential if there are elevations in ST or elevation in troponin levels. NCSBN Client Need Topic: Physiological Adaptation; Sub-Topic: Medical Emergencies Last Updated - 13, Nov 2021

You are educating a patient newly diagnosed with hypertension about sodium and its role in blood pressure. Which of the following statements about sodium are true? Select all that apply. A. Sodium cannot be completely eliminated from the diet. B. There is no sodium in fresh fruits and vegetables. C. Canned vegetables should be avoided. D. The body needs some sodium as it plays an important role in water balance. Submit Answer

Explanation Choices A, C, and D are correct. A is correct. It is not possible to eliminate sodium from the diet, nor would it be recommended. Sodium is a principal cation and it plays a role in driving the sodium-potassium pump as well as regulating water balance, so wholly eliminating sodium is not a good idea. C is correct. Canned vegetables do use a large amount of sodium to preserve flavor, so you should advise your patient with hypertension to avoid them. D is correct. The body indeed needs some sodium as it plays a vital role in water balance, so this is an appropriate teaching point for your patient. Choice B is incorrect. There is a small amount of sodium in fresh fruits and vegetables, but these are still good diet choices for a patient newly diagnosed with hypertension. NCSBN Client Need: Topic: Physiological Integrity, Subtopic: Physiological Adaptation, Renal Additional Info Source : Archer Review Last Updated - 19, Oct 2021

The new nurse understands that the high-quality cardiopulmonary resuscitation (CPR) for an adult consists of which of the following: Select all that apply. A. Compression rate of 100 to 120 per minute. B. Compression depth of 1.5 inches. C. Allow full chest recoil between compressions. D. Rotate compressor at least every 2 minutes. Submit Answer

Explanation Choices A, C, and D are correct. The purpose of CPR is to move blood through the heart and to the body's cells to prevent cell death. According to the American Heart Association (AHA), high-quality CPR includes a compression rate of 100-120 per minute to a depth of 2-2.4 inches. The provider must allow full chest recoil between each compressor. Full chest recoil allows the heart chambers to fill with blood between compressions. When the ventricles fill, more oxygenated blood will be available to the cells. Fatigue will result in less effective compressions, so the AHA recommends that the compressors rotate every 2 minutes or five cycles of compressions to prevent fatigue. Choice B is incorrect. The compression depth of 1.5 inches is inaccurate. Last Updated - 01, Aug 2021

The nurse is caring for a client who arrives to the emergency department (ED) complaining of chest pain radiating to the arm. The nurse should do which of the following? Select all that apply. A. Obtain an electrocardiogram (ECG) B. Prepare the client for prescribed cardioversion C. Establish intravenous (IV) access D. Insert an indwelling urinary catheter E. Administer prescribed nitroglycerin Submit Answer

Explanation Choices A, C, and E are correct. A client presenting with chest pain radiating to the arm warrants immediate intervention as it could be an acute myocardial infarction. The nurse is correct to obtain an electrocardiogram, establish intravenous access, and administer the prescribed nitroglycerin. Choices B and D are incorrect. Preparing a client for cardioversion is not necessary unless an applicable arrhythmia is present. Finally, inserting a urinary catheter is not required for an individual presenting with angina. Last Updated - 26, Oct 2021

The nurse has provided education to a client with atrial fibrillation. Which of the following statements by the client would require a follow-up? Select all that apply. A. "I have an increased risk for a stroke." B. "I should weigh myself daily at the same time." C. "I may be prescribed medications such as amiodarone." D. "I should wear a mask when I am in public." E. "I should follow-up with my primary healthcare provider (PHCP) if I develop shortness of breath." Submit Answer

Explanation Choices B and D are correct. These two statements indicate that the patient needs further follow-up education to correct the misconceptions. The client does not need to weigh themselves daily (Choice B) as that would be applicable for CHF and not for atrial fibrillation. Considering daily weight checks in CHF is useful to detect excess fluid retention, which may precede symptoms such as shortness of breath. Wearing a mask in public is unnecessary as an infection is not the concern here (Choice D). Choices A, C, and E are incorrect. These options are wrong because these statements indicate correct understanding by the patient and do not require follow-up teaching. Atrial fibrillation is a common dysrhythmia that results in a decrease in an atrial kick. A client with atrial fibrillation is at risk for an ischemic stroke (Choice A) because of the formation of clots in the atrial appendage. Treatment for atrial fibrillation range from medications (diltiazem, amiodarone) to cardiac ablation (Choice C). Finally, the client needs to notify the PHCP if they develop dyspnea because this could be an indication of atrial fibrillation with a rapid ventricular response (RVR), which requires immediate medical attention (Choice E). Additional Info Source : Archer Review Last Updated - 15, Feb 2022

The nurse is assessing a client who has developed cardiac tamponade. Which of the following findings would the nurse expect to observe? Select all that apply. A. Bibasilar crackles B. A systolic murmur C. Bradycardia D. Jugular Venous Distention E. Hypotension Submit Answer

Explanation Choices D and E are correct. Classic manifestations of cardiac tamponade include tachycardia, tachypnea, pericardial rub, jugular venous distention, and hypotension with a narrowed pulse pressure. Choices A, B, and C are incorrect. Bibasilar crackles, a systolic murmur, and bradycardia would not be consistent with cardiac tamponade. The client with cardiac tamponade would have tachycardia to increase cardiac output, coupled with a pericardial friction rub if the tamponade is caused by inflammation. NCLEX Category: Physiological Adaptation Activity Statement: Alterations in Body Systems Question type: Knowledge/comprehension Additional Info Cardiac tamponade may be caused by an array of infectious and noninfectious reasons. Immediate treatment of cardiac tamponade would be pericardiocentesis. A needle is inserted to aspirate the pericardial fluid in this ultrasound-guided procedure. The provider may elect to leave a temporary catheter in place in the pericardium to drain more fluid. Nursing care involves reporting any suspicion of cardiac tamponade promptly to the provider. Last Updated - 29, Oct 2022

The nurse enters the room of a client who appears unresponsive on the ground. The nurse should perform which actions? Place the actions the nurse should take in the appropriate order. Assess the client's carotid pulse and assess if the client is breathing Start chest compressions at 100 to 120/minute Notify the rapid response team (RRT) Shout for the code cart/defibrillator Provide rescue breaths Submit Answer

Explanation When a nurse arrives at a scene where a client is unresponsive, the nurse should immediately summon help by activating the RRT. Once the team has been called, the nurse should request the code cart/defibrillator come to the bedside. Having the defibrillator readily available will ensure that emergency medications and any countershock can be delivered without delay. The nurse should then assess the client to see if they are breathing and check for a carotid pulse for no more than ten seconds. If a pulse is not palpable, the nurse should start chest compressions at 100 to 120 per minute. Once thirty compressions have been administered; the nurse should provide two rescue breaths. Once an AED arrives, the nurse should apply the AED (or defibrillator) and determine if the client has a shockable rhythm, such as ventricular tachycardia without a pulse or ventricular fibrillation. Image Credit: The American Heart Association Additional Info When performing CPR, the nurse needs to minimize interruptions and focus on providing effective compressions and ventilations ✓ A compression rate of 100-120/minute is desired ✓ An AED should be made available as urgently as possible ✓ Assessing for a pulse should not take more than ten seconds ✓ When obtaining a pulse for an infant, the nurse should assess the brachial artery. For a child and adult, the nurse will use the carotid artery ✓ Immediate family members should be allowed to be present during resuscitation as this has promoted better grieving Last Updated - 20, Jan 2023


Conjuntos de estudio relacionados

End of Ch. 2 quiz - Types of Life Policies

View Set

Linear and Quadratic Systems and Direct and inverse Variation

View Set

Courts and Judicial Procedure Exam 1

View Set

Mental Health Exam 2, Ch. 4-6, 11, 12, 21, 35

View Set

Mental Health Ch. 25_Suicide and Nonsuicidal Self-Injury

View Set

UNCC Project Management Final Exam Review Guide Multiple Choice

View Set

Business Law Final Exam Vocabulary, Cases, and Questions

View Set